You are on page 1of 42

FEDERAL CIVIL PROCEDURE

After this lecture, you will be able to:


1. Select and evaluate the forum for suit by assessing subject matter jurisdiction, personal jurisdiction and
venue.
2. Determine the steps of proper service by process.
3. Understand the pleading of a case (notice pleading, plausibility pleading, heightened pleading, responses to
complaints, and amendment of pleadings).
4. Determine whether to join multiple claims and multiple parties to a lawsuit, including counterclaims, cross-
claims, third-party claims, and compulsory joinder.
5. Identify motions to dismiss at the pleading stage.
6. Identify and compare discovery tools, including interrogatories, requests for admission, and requests for
documents.
7. Evaluate viability of pretrial adjudication (dismissal, default, and summary judgment).
8. Assess the right to trial by jury.
9. Analyze trial and post-trial motions for judgment as a matter of law and for a new trial.

I. JURISDICTION AND VENUE

A. SUBJECT MATTER JURISDICTION


1. Subject matter jurisdiction, or SMJ for short, is the power of the court over a particular case. Federal
district courts get their SMJ from Congress and, for our purposes, have it in four different circumstances:
a. Federal question jurisdiction
b. Diversity jurisdiction
c. Supplemental jurisdiction
d. Removal jurisdiction
2. As we discuss each of these jurisdictional grounds, keep in mind that federal SMJ will always concern
whether a plaintiff may file a claim in the federal court system, or must instead seek relief in a different
court system (such as in state court). Thus, SMJ has nothing to do with the geographic location of the
lawsuit. Location only matters to questions of personal jurisdiction and venuetopics we will cover a bit
later.
3. Federal Question Jurisdiction (28 U.S.C. 1331)
a. The first type of claim that federal courts are empowered to hear are claims presenting a question of
federal law. Federal law includes the federal Constitution, federal statutes and treaties, as well as,
less commonly, federal common law. (We will discuss federal common law a bit later.) In addressing
federal question jurisdiction issues, there are two key ideas to remember.
b. Well Pleaded Complaint Rule (WPC)
(1) The WPC rule holds that federal question jurisdiction works only if the federal Q arises within the
plaintiffs affirmative claimregardless of any defense the defendant might raise.
EXAMPLE: P is passed over for a promotion by D, her boss. P believes that D passed her over
because she was a woman and thus discriminated against her on the basis of her sex. Federal
law bars such discrimination. P may sue D in federal court using federal question jurisdiction.
EXAMPLE: P is passed over for a promotion by D, her boss. P believes that D passed her over
because she was a woman and thus discriminated against her on the basis of her sex. P wishes
to bring suit against D for a violation of a state law barring sex discrimination. From discussions
with D, P knows that D believes that federal law immunizes it from liability under state law. P
anticipates this argument in her complaint and argues that federal law does not immunize the D.
P may not sue in federal court under federal question jurisdiction because federal law is not part
of her affirmative case; rather it is part of Ds defense. Note that it is immaterial here that:
(1) P wrote federal law into her complaint, and
(2) the federal court will very likely have to resolve a federal question in the suit.
c. Federal Question Inside State Law Claims
(1) Most of the time, a federal question action will be composed entirely of federal law. Sometimes,
however, federal question actions will involve state and federal law. This is permissible as long
as the federal question: (1) is actually in dispute, (2) demands the expertise of federal judges,
and ought to be resolved as uniformly as possible; and (3) is not so commonly present in state
law actions that federal courts would end up taking a large portion of state courts traditional
jurisdiction.
EXAMPLE: P rents land to D, a farmer. The lease states that D must comply with all federal
environmental laws. One day, P sees D dumping oil in a creek on the land, an action she
believes to be a violation of federal environmental law. D believes that dumping oil is permissible
in limited quantities. P wishes to terminate the lease in an action under state law. P can sue in

1
federal court because her state law claim will require the court to interpret federal environmental
law, federal judges are likely to have a comparative expertise at such law, such law ought to be
resolved as uniformly as possible, and the federal issues are not so common on landlord tenant
disputes that federal courts would end up taking the state courts landlord tenant jurisdiction
from them.
EXAMPLE: P rents land to D, a farmer. The lease states that D may not apply chemicals to the
land that are listed in the EPAs list of dangerous chemicals. P believes that D put Chemix on
the landa chemical on the dangerous chemicals list. D agrees that Chemix is on the list but
simply denies that he ever used Chemix. P may not sue in federal court using federal question
jurisdiction because there is no actual dispute pertaining to federal law.
4. Diversity Jurisdiction (28 U.S.C. 1332)
a. The second type of claim that federal courts are empowered to hear are so-called diversity claims.
b. A diversity action is a claim between citizens of different states (or between an American citizen and
an alien) for an amount over $75,000
c. Diversity Must Be Complete
(1) Complete diversity exists as long as no plaintiff and defendant are citizens of the same state.
EXAM TIP: The easiest way to figure this out is to make a list of the citizenship of every plaintiff
and then see whether any state is on both sides of the v.
EXAMPLE: Mary, Ken and Paul are injured in a bus accident. They join together as plaintiffs to
sue Joe, the bus driver. Mary is a citizen of Maryland, Ken is a citizen of Kentucky, Paul is a
citizen of Kentucky, and Joe is a citizen of Maryland. Complete diversity is not present here
because Mary and Joe are both from Maryland. Note that it is irrelevant that Ken and Paul are
both citizens of the same state.
(2) Timing: Diversity is measured at the time the lawsuit is filed. Thus, even if a party moves to a
different state after the lawsuit is filed, jurisdiction will not be affected. However, if a complaint is
amended to add or dismiss a party, this can affect jurisdiction.
EXAMPLE: Mary, a citizen of Maryland, sues Ken, a citizen of Kentucky, for $80,000 in federal
court. The court has subject matter jurisdiction. During the suit, Ken moves to Maryland. The
court still has subject matter jurisdiction. After Ken moves to Maryland, Mary decides to amend
her complaint to add Joe (a citizen of Maryland) as a defendant. Marys amendment will destroy
the courts subject matter jurisdiction.
EXCEPTION: In class actions where the class contains over 100 persons and the amount in
controversy exceeds $5,000,000, diversity need not be complete. It need only be minimal,
meaning that federal jurisdiction exists if any single member of the class is diverse from any
single defendant. We will touch upon this again later when we talk about class actions.
d. Determining Citizenship
(1) Human Beings
(a) Human beings can only be citizens of one state at a time. A human beings state of
citizenship is the state in which she resides and intends to remain indefinitely. Someone
who intends to remain indefinitely need not intend to stay forever; she simply cannot have
any present plans to leave.
EXAMPLE: P grew up in Colorado and currently attends college in Chicago, Illinois. She
has fallen in love with the city and no longer returns home during the summers. She has
obtained an Illinois drivers license, bought a small condominium, and joined several
community organizations. P is a citizen of Illinois for the purpose of diversity jurisdiction
because she resides there and, as evidenced by her activities, apparently intends to
remain there for the foreseeable future.
(b) Hobo Rule: A human beings citizenship does not change until she plants her roots in a
new state by residing there and intending to remain there indefinitely.
EXAMPLE: Honeysuckle Jim was born in Oklahoma but left home at 11 years old to ride
the rails. He is now 52 and has been riding the rails his entire life, though he has never
once passed through Oklahoma. Over the past month, hes been living in a boxcar in an
Omaha, Nebraska rail yard. He likes it there but said, Me? Oh, I dont live anywhere. Only
home I got is the road. Honeysuckle is still an Oklahoma citizen because he acquired that
citizenship at birth and has never resided anywhere else that he intended to remain
indefinitely.
(2) Corporations
(a) A corporation is a citizen of the state or states in which it is incorporated AND the state in
which it maintains its principal place of business
1) A corporations principle place of business is its nerve center, i.e. the place from
which the corporations senior officers direct the activities of the corporation
EXAMPLE: P is a citizen of Ohio. Last year, she suffered serious injuries in a
bicycle accident and wants to sue the manufacturer, InMotion Bikes, Inc. InMotion

2
is incorporated under the laws of Delaware and headquartered in Ohio. P cannot
sue InMotion in federal court under diversity jurisdiction because InMotion is a
citizen of Ohio. It is irrelevant that InMotion is also a citizen of Delaware. The
presence of Ohio on both sides of the v is enough to destroy diversity.
EXAMPLE: Assume that InMotion manufactures its bikes in Ohio but runs its
business from its corporate offices in Greenwich, Connecticut. P and InMotion are
now diverse because In Motions manufacturing plant is not its nerve center.
InMotion thus has no Ohio citizenship.
(3) Unincorporated Associations
(a) If a party is not a person or a corporation, it is an unincorporated association. Examples of
these entities include unions, limited liability companies (LLCs), partnerships, and ordinary
sole proprietorships. The citizenship of these entities is the citizenship of every member.
Thus, unincorporated associations can potentially be citizens of all 50 states.
EXAMPLE: Mary Louise and her three siblings inherited farmland from their parents.
Instead of splitting the land into 4 separate plots, they formed Farmland LLC, a company
that owns and maintains the land. Each sibling is a joint owner of the LLC. Mary Louise
lives in Maryland, two siblings live in Missouri, and another lives in Kansas. The LLCs
citizenship is Maryland, Missouri, and Kansas.
EXAMPLE: The Teamsters is a large union of workers in various trades. The union is so
large in fact that it has members in every state in the union. The Teamsters will thus never
be diverse from an American citizen. They could, however, be diverse from an alien.
(4) Representatives
(a) Sometimes the party in interest is not able to litigate on his or her own behalf, whether
because the party is a minor, deceased, or otherwise lacks capacity. In these instances,
litigation occurs through personal representatives. For the purpose of diversity jurisdiction,
however, the key inquiry is the citizenship of the incapacitated party, not the citizenship of
the representative
e. Amount in Controversy
(1) Not only must there by complete diversity for diversity jurisdiction to be available, the amount in
controversy must exceed $75,000.
(a) A court will defer to the plaintiffs allegation of her financial injury unless it appears to a
legal certainty that such allegations are incorrect.
(b) Additionally, it is immaterial whether the plaintiff actually recovers over $75,000 in
damages as long as an appropriate amount was pled in good faith at the outset.
EXAMPLE: P suffered a black eye in a bar brawl. She sued D for $76,000, including
$10,000 compensatory damages and $66,000 in punitive damages. Although these
damages might seem high for a black eye, the court will almost certainly defer to the
plaintiffs allegation of harm and find the amount in controversy exceeds $75,000.
EXAMPLE: P suffered a broken back in a car accident. She sued D (who was diverse from
her) for $80,000 for her injuries. The case went to trial and P only recovered $50,000.
Jurisdiction was appropriate because P sought $80,000 in damages at the outset.
EXAMPLE: P hired D to perform some services for $10,000. D breached the contract, and
P sued D for $76,000, including $10,000 in compensatory damages and $66,000 in
punitive damages. The amount in controversy here is only $10,000 because, under state
law, punitive damages are not recoverable in breach of contract actions.
(2) Aggregation
(a) If a particular plaintiff brings multiple claims against a particular defendant, a court will
aggregate the amount in controversy for each of those claims to determine whether a
sufficient amount in controversy exists.
(b) If the plaintiff brings multiple claims against multiple defendants, aggregation is
impermissible.
EXAMPLE: P has had it up to here with D. Last year, D ran over Ps cat (causing her
$30,000 in damages). Then, a few months later, D cut down a tree that fell on Ps house
(causing her another $30,000 in damages). And just last week, D defamed P by calling her
a lying, cheating, son of a gun (causing her $20,000 in damages). If P sues D for all three
claims at once, the amount in controversy for diversity jurisdiction will be $80,000.
EXAMPLE: P was in a bar when Dip and Dup beat him up. He suffered $50,000 in injuries.
If P sues Dip and Dup in the same suit, the amount in controversy will be $50,000,
regardless of the fact that P is alleging that each defendant, pursuant to the rules of joint
and several liability, is personally liable for $50,000. The same answer would be obtained if
two plaintiffs each sued a single D for $50,000 in a single suit. In that suit, D may have to
pay out $100,000 in damages, but the amount in controversy for each P is only $50,000.

3
5. Supplemental Jurisdiction (28 U.S.C. 1367)
a. If a federal court lacks federal question or diversity jurisdiction over a claim, it can sometimes use
supplemental jurisdiction to hear the claim. Very generally, supplemental jurisdiction allows a claim
falling outside FQ or diversity jurisdiction to piggy back on a claim that does fall within one of those
jurisdictions
(1) Supplemental jurisdiction can be complicatedif you let it. If you follow the steps below, you can
master this area without too much pain and suffering.
b. Step 1: Determine Relatedness
(1) Does the claim seeking piggy back jurisdiction (the piggy back claim) arise from the same
transaction or occurrence as another claim falling within federal question or diversity (the
anchor claim)? If the claim does arise out of the same transaction or occurrence, go to Step 2.
If the claim does not, piggy back jurisdiction is unavailable.
c. Step 2: Watch for Sneaky Plaintiffs
(1) A sneaky plaintiff is one who is attempting to circumvent the limitations on diversity jurisdiction.
Thus, the first thing to remember here is that, if the anchor claim is a federal question claim,
there is not concern with sneakiness and you should go straight to Step 3.
(2) In contrast, if the anchor claim is a diversity claim, you need to determine whether the potential
piggy back claim is brought by a P against a party whose presence in the suit is pursuant to
Rule 14, 19, 20, or 24. If it is, then there is reason to think that the plaintiff may be attempting to
sneak claims into a suit that couldnt be there if P had filed them at the outset.
(a) Supplemental jurisdiction is thus not available for claims by Ps against parties joined in this
way. If the claim is not by a P against such a party, however, then go to Step 3.
(3) We will cover the Federal Rules later in this outline, but for now, you can spot the use of these
rules in the following ways:
(a) Look for third party defendants. Rule 14 allows parties defending a claim to force another
party to join the lawsuit to share in the potential liability. Thus, if P sues Doctor for medical
malpractice, Doctor can use Rule 14 to implead Nurse who, in Doctors opinion, should share
in any liability imposed on him because Nurse was partially negligent. The suit would be
diagrammed like this:

P v. Doctor v. Nurse

1) Nurse is a third party defendant and P cannot rely on supplemental jurisdiction to sue
Nurse if the anchor claim is a diversity claim. P can rely on diversity or federal
question jurisdiction to sue Nurse, however.
(b) Look for stacked parties. Rules 19, 20, and 24 allow multiple parties on the same side of
the v to sue or be sued together. (The differences between these Rules are unimportant
here.) Thus, if P gets beat up by Dip, Dup, and Dop, he can sue all three in the same suit.
The suit would be diagrammed like this:

v. Dip
P v. Dup
v. Dop

1) Dip, Dup, and Dop each is a stacked party, and P cannot reply on supplemental
jurisdiction to sue any of them if the anchor claim is a diversity claim. P can rely on
diversity or federal question jurisdiction to sue Dip, Dup, or Dop, however.
d. Step 3: Consider State Prerogatives
(1) Even if Steps 1 and 2 are satisfied, is there nonetheless a good reason for the court to decline
piggy back jurisdiction? The reasons provided in federal law are all based on the belief that
federal courts ought not to reach out and decide questions of state law in certain instances, thus
respecting states traditional prerogative to decide matters of state law
(2) Good reasons include:
(a) the piggy back claim involves a novel and complex issue of state law;
(b) the piggy back claim substantially predominates over the federal question or diversity
claim;
(c) the court has dismissed the anchor claim; and
(d) in exceptional circumstances, where other compelling reasons exist.
(3) If a good reason exists, a court may decline to exercise supplemental jurisdiction, but is not
required to do so.

e. Review of Steps 1, 2, and 3


4
EXAMPLE: P bought a car from D, who tampered with the odometer before the sale. P sued D for:
(1) a violation of federal trade law with damages of $1,000, and (2) common law fraud with damages
of $5,000. The court has subject matter jurisdiction over the first claim because it is a federal
question. The court has subject matter jurisdiction over the second claim because it is related to a
federal question claim, and there are (apparently) no reasons to decline piggy back jurisdiction.
EXAMPLE: P bought a car from D, who tampered with the odometer before the sale. P sued D for a
violation of federal law (damages: $1,000) and common law fraud (damages: $5,000). In the same
suit, P also sued Car Co., Ds employer, on the same claims and for the same damages. The court
can hear both federal question claims (one against D and one against Car Co.) and both fraud claims
(one against D and one against Car Co.). Although the piggy back claims here are brought against
stacked defendants, this is unimportant where the anchor claim is a federal question.
EXAMPLE: P bought a car from D, who tampered with the odometer before the sale. P is a citizen of
Ohio, and D is a citizen of Missouri. P sued D for common law fraud, alleging $80,000 in damages. D
tampered with the odometer at the instruction of his employer, Car Co., and thus impleaded Car Co.
(headquartered in Ohio and incorporated in Delaware) to share in Ds potential liability. Ds claim
against Car Co. was for $50,000. The court has subject matter jurisdiction over Ps fraud claim under
diversity of citizenship and, assuming there are no Step 3 problems, has subject matter jurisdiction
over Ds contribution claim against Car Co. under supplemental jurisdiction. The claim is brought by
D, not P, and therefore does not fail Step 2 of the analysis.
EXAMPLE: Same facts as the previous example, but assume that after Car Co. has been impleaded,
P decides to file some claims against Car Co. as well. She files a federal claim against Car Co. for
$1,000 as well as a common law fraud claim for $80,000. The court has SMJ over the federal claim
pursuant to its federal question jurisdiction. The court does not have SMJ over the fraud claim
because the parties are not diverse (thus foreclosing diversity jurisdiction) and because the fraud
claim: (1) is based on a diversity anchor claim, and (2) is a claim by a P against a third party
defendant (thus foreclosing supplemental jurisdiction).
EXAMPLE: P, a citizen of Texas, sues D1, a citizen of Florida, for $80,000 in a state law action.
During discovery, P learns that D2 (also from Florida) committed his own harms against P that
amounted to $50,000. D2s actions were part of the same occurrence underlying D1s actions. P
amended her complaint to add D2 as a defendant. The court can hear Ps claim against D1 pursuant
to diversity jurisdiction but cannot hear her claim against D2. The court has no diversity jurisdiction
because the amount in controversy does not exceed $75,000, does not have federal question
jurisdiction because the claim does not present a federal question, and does not have supplemental
jurisdiction because the claim is: (1) based on a diversity anchor, and (2) by P against a stacked
party.
EXAMPLE: P1, P2, and P3 (all citizens of Kansas) were each injured in a bus accident. P1 suffered
$80,000 in damages, and P2 and P3 each suffered $50,000 in damages. P1, P2, and P3 joined
together to sue D, the bus company, each pressing his own common law claim against D. D is a
citizen of Maryland and Delaware. The court has diversity jurisdiction over P1s claim against D. With
regard to P2s and P3s claims, the court does not have diversity jurisdiction because the amount in
controversy is insufficient and does not have federal question jurisdiction because there is no federal
question. However, the court does have supplemental jurisdiction because the claims are related to
P1s anchor claim, the claims are not by a P against a stacked party (they are by a stacked party
against a single D, not against a stacked party), and there are no apparent Step 3 problems.
6. Removal (28 U.S.C. 1441, 1446)
a. Removal jurisdiction permits a federal court to hear a case that was originally filed in state court.
Removal is only an option for defendants; plaintiffs cannot remove a suit to federal court after having
chosen state court.
b. Grounds
(1) A defendant can remove a case from state to federal court if the case could have originally been
filed in federal cour
(2) Thus, in determining whether removal is permissible, the federal court simply determines
whether it would have had subject matter jurisdiction over the suit (using federal question,
diversity, and supplemental jurisdiction) had the plaintiff started in federal court to begin with.
EXAMPLE: P sues D in Arizona state court for a violation of federal anti-discrimination law. D
may remove the suit to federal court because the plaintiff could have filed her suit in federal
court as an initial matter.
NOTE: Counterclaims, cross claims, and impleader claims (we will discuss these later) are not
sufficient, on their own, to permit removal.
EXAMPLE: P sues D, who is diverse from P, in state court for $50,000 in a state law action. D
counterclaims with a state law claim for $80,000. D may not remove the suit, even though the
counterclaim on its own could have been filed in federal court.
c. Exception

5
(1) A defendant may not remove a case if:
(a) federal jurisdiction would be grounded only in diversity jurisdiction, and
(b) the defendant is a citizen of the state where the plaintiff filed suit
(2) This exception could be called the home-state defendant rule. Note that the home-state
defendant rule is inapplicable in certain types of class actions, a point we will discuss later.
EXAMPLE: P, a citizen of Michigan, travels to Ohio for vacation. While there, she gets into a car
accident with D, a citizen of Ohio. P sues D in Ohio state court for $80,000. Although the parties
are diverse and the amount in controversy exceeds $75,000, D may not remove the case to
federal court because jurisdiction there would be grounded only on diversity, and D is a citizen of
Ohio.
d. Multiple Defendants
(1) Removal is not permitted unless all defendants agree to removal. Thus, any defendant wishing
to stay in state court can force the case to remain there by not joining in a notice of removal filed
by another defendant. Like the home-state defendant rule, this rule is also relaxed in certain
types of class actions. We will thus return to this point again later.
e. Timing
(1) In General: A defendant must remove within 30 days of when the grounds for removal become
apparent
(a) Normally, the ground for removal will be apparent when the defendant is served with
process because service will include a copy of the complaint, which will make clear
whether federal question or diversity jurisdiction is available. Sometimes, however, if the
plaintiff amends her complaint to add a claim, change the amount in controversy, or drop a
party, removability will become clear well after service of process.
(2) Multiple Defendants Served at Different Times: If P sues multiple defendants and serves
them with process at different times, the 30-day window for removal runs from the moment the
final defendant is served. Thus, even if the first defendant served with process declines to
remove the suit within 30 days of her service, she is entitled to join in removal if a later-served
defendant elects to remove. Later service on other defendants does not, however, empower the
first-served defendant to remove on her own accord; she may only join in a removal initiated by
a later served defendant.
(3) Special Rule for Diversity Actions: If the removal to federal court would be based on diversity
of citizenship, the defendant must remove the case within 1 year of the day it was filed (not
served), unless the plaintiff attempted to thwart removal in some way
(a) This one-year limitation will come into play when a plaintiff changes her claims or parties
after initially filing her suit and having process served.
EXAMPLE: P sued D for injuries she suffered in a car accident. P and D are diverse, and
the amount in controversy is $80,000. Sixty days after the suit was filed, but only 20 days
after he was served with process, D files a notice of removal. D may remove the suit
because the 30-day window for removal begins to run when he was served with process.
EXAMPLE: P sues D1 and D2 in state court on May 1. The suit is removable. She has D1
served with process on June 1, but D1 declines to remove the suit within the 30-day
window. P then has D2 served with process on July 1. D2 elects to remove the suit on July
10. D1 can join in removal at that point, even though D1 was served with process more
than 30 days beforehand. Note that if D2 declines to remove the suit, D1 may not attempt
to remove it within 30 days of when D2 was served. D1 is stuck with D2s decision not to
remove.
EXAMPLE: Michael suffered injuries at the hands of Billy and Linda. Michael is a citizen of
New York, Billy is a citizen of Oklahoma, and Linda is a citizen of New York. Michael sued
Billy and Linda in New York state court for $80,000. Thirteen months later, Michael and
Linda agreed to settle, and Michael therefore dismissed his claim against her. Although
Michael and Billy are completely diverse, Billy may not remove because more than one-
year has elapsed since Michael filed suit. If Michael named Linda solely to thwart removal
(rather than because he thought she was liable), Billy can remove the suit.
EXAMPLE: P, a citizen of Illinois, sued D, a citizen of Utah, for $50,000 in state court in
Illinois. P actually suffered $80,000 in damages, but doesnt want to litigate the suit in
federal court. Six months after filing suit, P amends her complaint to add an additional
$30,000 in damages. D can remove the suit to federal court within 30 days of receiving the
amended complaint. Suppose P amends her complaint in the same way, but does so
thirteen months after filing suit. D can only remove at this point if he can show that Ps
initial filing and later amendment were an effort to thwart removal.

B. PERSONAL JURISDICTION

6
1. Personal jurisdiction, or PJ for short, is the power of a court over a particular party. PJ focuses on the
state in which the suit has been filed. To determine whether a federal court has PJ over a particular party,
you need to analyze two bodies of law:
a. The PJ law of the state in which the federal court sits, and
b. The PJ law within the United States Constitution
2. Step 1: Look at State Law
a. To determine whether it may assert PJ over a party, the first question a federal court must ask is
whether a state court in the same state would assert PJ over that party.
(1) If the state court would assert PJ, then the federal court may do so as well (subject to the
constitutional considerations discussed below).
(2) If the state court would not assert PJ, then the federal court may not assert PJ.
NOTE: This rulelook at state law firstis from Congress and applies in 99.9% of cases.
Congress need not have chosen this rule, however, and in rare circumstances, Congress has in
fact taken this approach. In a few circumstances, Congress has authorized the federal courts to
assert personal jurisdiction over a defendant without regard to state law. Its not important to
know all of these circumstances (though we will cover one of them later in the outline), but it is
important to know that Congress, if it desired, could subject a defendant to personal jurisdiction
in federal court without regard to state law.
EXAMPLE: P sues D in the Southern District of New York for punching him while the two were visiting
New York City from out of state. The New York legislature has authorized its courts to assert PJ over
defendants that commit a tort while in the state. Because a state court in New York could assert PJ
over D, Step 1 has been satisfied. That is, the New York federal court may assert personal jurisdiction
over D, subject to constitutional constraints.
EXAMPLE: P purchased a trinket from Trinkets, Inc. while visiting New York City. She took it home to
Oregon where she was injured by it. She sued Trinkets, Inc. in federal court in New York. Assume that
New York State has a law that bars courts from asserting PJ over a defendant if the alleged injury
occurred outside New York. Because a New York state court may not assert PJ over Trinkets, Inc.,
Step 1 has not been overcome. That is, the New York federal court may not assert personal
jurisdiction over D regardless of any constitutional considerations.
3. Step 2: Look at the Fourteenth Amendment
a. Just because state law permits PJ over a particular party does not mean that the federal court may
automatically assert PJ. The federal court must also determine whether the state law is constitutional
under the Due Process Clause of the Fourteenth Amendment. A state law will be constitutional if it
authorizes PJ in one of five circumstances.
(1) Well discuss these circumstances below, but for short, just remember this: PJ Rarely Causes
Severe Mental Strain.
(a) Residency
(b) Consent
(c) Service
(d) Minimum Contacts
(e) Substantial Business
NOTE: Before digging into these five circumstances, keep in mind that many states authorize PJ to the full
extent of the Fourteenth Amendment. If a state does that, all you need to do is determine whether PJ over
a particular defendant is permissible under the Constitution. If it is, then both state law and constitutional
law are satisfied.
b. Residency
(1) If a party is a resident (a.k.a. domiciled) in the state where the lawsuit has been filed, PJ over
the defendant is constitutional. Human beings reside in the states where they live. A corporation
is a resident of the state(s) where it is incorporated as well as the state where it is
headquartered. Other entities are residents of the state in which they maintain a headquarters or
principal place of business.
EXAMPLE: P lives in Utah but drove to Nevada for a weekend getaway. While in Nevada, P got
into a fight with D, a resident of Nevada. If P sues D in Nevada federal court, the court will have
PJ over D as long as state law authorizes its own courts to hear allegations against Nevada
state residents (which it almost certainly does). Such a grant of jurisdiction comports with the
Fourteenth Amendment.
EXAMPLE: P performed work for D in Illinois. D is a non-profit organization headquartered in
Ohio. D is a resident of Ohio for the purpose of personal jurisdiction, and PJ over it in Ohio is
thus constitutional.
c. Consent
(1) If a party consents to the federal courts PJ over him, PJ is constitutional. Consent generally
occurs in one of three ways:
(a) Appearance: When a party appears in court without objecting to personal jurisdiction, the

7
party has consented to the courts power. The party must object to PJ in its initial filing or
appearance before the court.
NOTE: This is how plaintiffs are most often subject to PJ. By asking the court to resolve a
dispute, the plaintiff essentially consents to the courts PJ over her. This means that the
vast majority of PJ questions will only pertain to defendants. PJ issues over plaintiffs only
arise in class action situationssomething we will get to later.
(b) Contract: Parties to a contract often put a choice-of-forum clause in their contractsa
clause that specifies the forum in which any lawsuit arising from the contract must be
litigated. If a defendant signs a contract with a choice-of-forum clause she has consented
to the power of the court specified in the clause
(c) Appointment: In exchange for the right to do business in a state, some state laws require
businesses to appoint agents located within the state to receive service of process if a
legal dispute arises. If a business (or other registered organization) appoints such an
agent, and the state law defines such appointment as consent; the business has
consented to the PJ of the federal court.
EXAMPLE: P, a resident of Rhode Island, went to Oklahoma for a rodeo. While there, he
was injured by D, a resident of Oklahoma. P sued D in Rhode Island federal court. D
traveled all the way to Rhode Island, appeared in court, and filed a motion arguing that the
statute of limitations had expired. The court denied this motion, and then D filed a second
motion challenging the courts personal jurisdiction over him. Assuming that state law
permits PJ over defendants in this situation, the federal court has PJ over D because D
consented by failing to challenge PJ in his initial filing.
d. Service
(1) If a defendant is served with process while in the state where the lawsuit has been filed, a
courts exercise of PJ over the defendant is constitutional. This type of PJ is sometimes called
tag because jurisdiction is obtained simply by tagging the defendant with process.
EXAMPLE: P, a resident of Wisconsin, travels to Ohio for vacation. While there, she is injured
by D in a boating accident. P wants to sue D for her injuries but doesnt want to travel back to
Ohio for the lawsuit. Luckily for P, D happened to be attending a conference in Milwaukee, and P
had him served while he was at the conference. As long as state law permits PJ over out-of-
state defendants who are served in-state, D is subject to PJ in Wisconsin because PJ in that
circumstance is constitutional.
(2) In the above example, PJ would not be constitutional if:
(a) P coaxed him into the state under false pretenses, or
(b) D was there to participate in a legal proceeding
e. Minimum Contacts
(1) The term minimum contacts is actually one partthe most popular part, to be sureof a
three-part test. For jurisdiction to be constitutional under a minimum contacts analysis, three
conditions must be satisfied:
(a) the defendant has established a minimum contact with the forum state,
(b) the claim against him arises from that contact, and
(c) jurisdiction will not offend traditional notions of fair play and substantial justice
(2) Minimum Contacts: A defendant has minimum contacts with a particular state if he causes
harm in the state, does business in the state, or has an interest in real property in the state.
Importantly, the defendant does not have to be in the state to cause the harm, do the business,
or have a property interest, but his contact with the state has to be purposefully established.
(3) Arising From: A lawsuit arises from a defendants contacts if the contacts played a role in
causing the lawsuit.
NOTE; None of the other grounds for personal jurisdiction (residency, consent, service, or
substantial business) have this requirement.
(4) Fair Play and Substantial Justice: To determine whether an assertion of jurisdiction is
consistent with notions of fair play and substantial justice, a court should consider the forum
states interest in providing redress to its own citizens, the plaintiffs interest in obtaining redress
for her harm, the states shared interests in efficiency and other important social policies, such
as whether the forum is so inconvenient that the defendant cannot adequately defend himself.
(a) The fair play factors are a lot to remember. And it may not even be worth it to remember
the entire list because the Supreme Court has not clarified the meaning of these phrases.
One thing that may help is to remember the facts of Asahi Metal Industry Co. v. Superior
Court, a case you likely read in Civil Procedure. Jurisdiction failed in Asahi because of the
fair play factors. There was no fair play in Asahi because the suit concerned a contract
between two foreign corporations that was entered into and executed outside the forum
state, and was controlled by the foreign law. Without a citizen to protect or a state law to
enforce, the forum state could not justify personal jurisdiction under the Constitutioneven

8
though minimum contacts and arising from were arguably established.
EXAMPLE: D, a resident of North Carolina, was driving through Rhode Island on his way
to Boston when he struck P with his car. In a subsequent suit in Rhode Island, D is subject
to personal jurisdiction because he caused harm in the state, the lawsuit arises from those
contacts, and Rhode Island has an interest in protecting its citizens from negligent drivers
and maintaining order on its roads.
EXAMPLE: P heard from a friend that D, a company in Nebraska, makes great custom
tailgates for pickup trucks. P, a resident of Kansas, called D and asked to order one. D
does all its business in Nebraska but agreed to make P a tailgate and send it to him. P
received the tailgate but couldnt install it because it was designed incorrectly. P can sue D
in Kansas because D did business in the state, the lawsuit arose from that activity, and
Kansas has an interest in protecting its citizens from breaches of contract.
EXAMPLE: P, a citizen of Texas, was driving in Connecticut when she got into a car
accident with D. D is a Connecticut citizen, but spends six weeks each summer in Texas
with his grandmother (and has been doing so for the past 10 years). P cannot sue D in
Texas because his contacts with the state did not give rise to the lawsuit. (P could,
however, wait until D comes back into the state next summer and have him served then.)
EXAMPLE: Motor MFG manufactured an electric motor in Michigan and sold it to Golf Cart
Co. in Ohio, who then installed it in a golf cart. P, a resident of Georgia, purchased the cart
over the phone, and Golf Cart Co. then shipped it from Ohio to P at home. P was later
injured when the carts electric motor caught fire, and he wants to bring suit in Georgia.
Golf Cart Co. will be subject to personal jurisdiction in Georgia because it transacted
business in the state or caused harm in the state, the suit arises from those acts, and fair
play concerns are not implicated. In contrast, Motor MFG will not be subject to personal
jurisdiction in Georgia because it did not purposefully establish any contacts with the state
that gave rise to Ps suit. Even if Motor MFG had sent other motors to the state in the past,
PJ is not permissible because those contacts did not give rise to Ps claim.
f. Substantial Business
(1) The fifth and final way in which a court can constitutionally assert personal jurisdiction over a
defendant is if it is doing substantial business in a particular state. The Supreme Court has
made clear that substantial business is a very high bar to clear. The business must be so
significant that the company, though not incorporated or headquartered in the state, is
essentially at home there.
EXAMPLE: Steel Co. is in the business of manufacturing and selling steel. It is headquartered in
New York City and incorporated in Delaware but conducts all of its operations in Baltimore.
There, it maintains a massive manufacturing and shipping operation that employs 60,000
persons. P was an employee of Steel Co. in New York City until she was fired last year. She
moved back in with her parents in Annapolis, Maryland and now wishes to sue Steel Co. for
unlawful termination. She may sue Steel Co. in Maryland because the company is essentially at
home in the state.
EXAMPLE: P was vacationing in Utah when she slipped and fell in a 7-11 convenience store.
After returning home to Arizona, P desired to bring suit against 7-11 in Arizona. 7-11 operates
131 stores in the state, roughly 8% of its total operations. 7-11 is not subject to personal
jurisdiction in the state because its business, though significant, is not so significant that Arizona
is essentially its home.

C. VENUE (28 U.S.C. 1391)


1. Like PJ, venue addresses geographical considerations. Unlike PJ, however, the law of venue dictates not
the state in which a suit can occur, but the region within a particular state where a suit can occur. In federal
court, this region is called a judicial district. Thus, to review: SMJ law tells you whether you can sue in the
federal court system; PJ law tells you the state where you can sue; and venue law tells you the particular
judicial district where you can sue.
2. The Basic Rules
a. The appropriate venue can be dictated by either:
(1) the residency of the defendant(s) or
(2) the location of the events giving rise to the suit, or both
b. Residency
(1) Where all defendants reside in the same state, the plaintiff may lay venue in the judicial district
where any single defendant resides.
NOTE: It may be helpful to recall the different situations in which a partys residency has been
relevant to our work thus far. Check out the following chart.

9
DOCTRINE WHERE RESIDENCY FITS IN RESIDENCY DEFINED

Subject matter A federal court has subject matter Natural persons are citizens of the state in which
jurisdiction; jurisdiction over any claim between they reside and intend to remain indefinitely.
Diversity citizens of different states where the Corporations are citizens of the state(s) in which
amount in controversy exceeds they are incorporated and maintain their principal
$75,000. place of business.
Other entities are citizens of the states of which all
its members are citizens.

Personal A court has personal jurisdiction over Natural persons are residents (or domiciles) of the
jurisdiction any defendant that is a resident of (or state in which they reside and intend to remain
domiciled within) the state where the indefinitely.
court sits. Corporations are residents of the state(s) in which
they are incorporated and maintain their principal
place of business.
Other entities are residents of the state in which it
maintains its principal place of business.

Venue Venue is proper in a judicial district Natural persons are residents of the state in which
where any defendant resides, as long they reside and intend to remain indefinitely.
as all reside in the same state. Corporations are residents of any judicial district
in which they are subject to personal jurisdiction.
Other entities are residents of any judicial district
in which they are subject to personal jurisdiction.

(2) The residency of human defendants is straightforward. Such defendants reside where they are
domiciled, or more simply, where they live. All other defendants (whether corporations,
unincorporated associations, partnerships, etc.) reside in every district in which they are subject
to personal jurisdiction for that district.
EXAMPLE: Kelly sued Gina and Tom for injuries she suffered in a car accident. Gina lives in
Cincinnati, OH and Tom lives in Cleveland, OH. Kelly may lay venue in either the Southern
District of Ohio (which embraces Cincinnati) or the Northern District of Ohio (which embraces
Cleveland) because Gina and Tom both live in the same state.
EXAMPLE: David was struck by a UPS truck in Cincinnati, OH. He wants to sue Jared, the
driver of the truck, for negligence, as well as UPS for Jareds negligence under the law of
respondeat superior. Jared is a citizen of Cleveland, OH, and UPS is headquartered in NY and
incorporated in DE. David can sue in the Northern District of Ohio or the Southern District of
Ohio. Both defendants reside in the state, though in different districts: Jared in Cleveland
because that is where he physically lives, and UPS in Cincinnati because that is where UPS is
subject to personal jurisdiction for this suit (based on minimum contacts).
c. Location of Events
(1) If the defendants reside in multiple states, venue will be dictated by the location of the harm.
(2) A plaintiff can also lay venue in a judicial district in which a substantial part of the events or
omissions giving rise to the claim occurred, or a substantial part of property that is the subject of
the action is situated.
NOTE: Keep in mind that the substantial events giving rise to a lawsuit can happen in more
than one judicial district.
EXAMPLE: Roger is driving through Kansas on his way home to California when he collides
with John and Nora. John is on his way home to Illinois, but is planning on dropping Nora off at
her home in Missouri on the way. If Roger wants to sue John and Nora, he can lay venue in the
district of Kansas because that is where a substantial event occurred. Note that Roger cannot
lay venue based on the residence of John and Nora because they do not reside in the same
state.
EXAMPLE: Tim, a resident of Maryland, bought a gadget from Whiz Gadget Co. The gadget
malfunctioned while Tim was using it at home, and he lost an eye. Tim wants to sue Whiz for
negligently manufacturing the gadget. Whiz is headquartered in OH, incorporated in DE, and
manufactures all its gadgets in Idaho. Tim can sue Whiz in the appropriate districts in Maryland
(where the injury happened, a substantial event), in Idaho (where the gadget was poorly made,
another substantial event), and in Ohio and Delaware (where Whiz resides).

10
3. Transferring Venues
a. Sometimes, after a lawsuit has been filed in a particular venue, there are reasons that it should be
transferred to a new venue. These reasons include the following:
(1) Convenience
(a) A court may transfer a case to a different federal venue if the case could have been filed
there in the first place (taking account of SMJ, PJ, and venue), and the transfer is
necessary for the convenience of the parties and/or witnesses.
(b) Note that transfer is a tool for moving a case within the federal courts; federal courts
cannot transfer cases to other court systems (like state court or foreign courts).
(2) Agreement
(a) If all parties join in a request to transfer a case to a particular venue, the court may transfer
the case to that venue, even if PJ and venue would otherwise be lacking. By joining in a
request for a transfer, the defendants have essentially consented to PJ and venue.
Importantly, if all parties join in a request, the court need not determine that the current
venue is inconvenient.
(3) Interest of Justice
(a) If a plaintiff files suit in an improper venue, the court may dismiss the suit or, in the interest
of justice, transfer the case to a venue where the case could have been filed originally.
EXAMPLE: Rita was injured in a bus crash on a back road in Montana. She believes that
Roadway, the bus company, is at fault. Roadway is incorporated in New York. Rita brought
suit in New York, and Roadway sought a transfer to district court in Montana. Roadway
argued that the suit will involve many witnesses (all of whom live in Montana), inspection of
the wrecked bus (which is in Montana), as well as visits to the scene of the accident
(again, in Montana). Because it would be much more convenient to litigate the suit in
Montana than New York, and because PJ and venue would be appropriate in Montana, the
court may order a transfer.
4. Forum Non Conveniens
a. Transfer assumes that there is another federal court that the case can be transferred to. Sometimes,
however, the most convenient court in which to litigate a suit is not in the United States. In these
circumstances, the federal court cannot transfer the suit; the court can only dismiss the suit without prejudice
so that the plaintiff may refile the suit abroad if she desires.
(1) Although forum non conveniens appears to be a venue concept, dismissals based on this
doctrine are actually dismissals for lack of subject matter jurisdiction.
b. The factors courts will consider in dismissing for forum non conveniens are the same as those
considered in transfer. Thus, the key difference between transfer and forum non conveniens is the
judicial response to the inconvenience. In the former, the court moves the case to a new place, and,
in the latter, the court dismisses the suit so that the plaintiff can move the case to a new place.
c. Finally, note that the court, in dismissing a case under forum non conveniens, will often condition the
dismissal on the defendant waiving any future challenges (should the case be re-filed in a different
country) to statute of limitations or other similar defenses.
EXAMPLE: Assume the same facts as in the bus accident above, but that accident occurred in
Canada. In that case, the federal court would likely dismiss the action so that the plaintiff, if she
desires, may re-file the suit in a Canadian court.
II. THE LAW APPLIED BY FEDERAL COURTS

A. FEDERAL QUESTION CASES


1. Ordinary Federal Question Cases
a. Most federal question cases that come before a federal court will simply involve a federal statute or
constitutional provision that has been violated. In such cases, the courts job is simple: just apply the
text of the federal law as well as any authoritative precedent interpreting that text.
2. Embedded Federal Question Cases
a. Recall from above that there are rare instances in which the success or failure of a state law claim
depends on a question of federal law. When a federal court hears this type of case, the court simply
applies state law to the issues pertaining to state law and federal law to the issues pertaining to state
law.
3. Federal Common Law Cases
a. Although most federal law has its roots in federal statutes or constitutional provisions, some federal
law exists simply as a body of precedent repeatedly applied and molded by the courts. This body of
law is known as federal common law and is generally confined to the following fields of law:
(1) maritime law
(2) foreign relations
(3) commercial rights and liabilities of the federal government
(4) property rights and liabilities of the federal government
11
b. When a case implicates one of these areas, the federal courts will apply (and, if need be, create) any
existing federal common law and ignore any state law that might otherwise apply.
EXAMPLE: The U.S. Treasury issued a check that was stolen and fraudulently cashed at a bank. The
bank has asked the federal government to pay it the funds that it paid out to the fraudster. If the check
was issued by a private individual, state law would control the check writers obligation and require
the check writer to pay the funds. Because this case concerns the commercial rights and obligation of
the federal government, however, the federal courts have the power to craft federal common law that
will preempt any contrary state law. The federal courts may craft a rule that relieves the federal
government of any obligation to pay on the check.

B. DIVERSITY CASES: ERIE ANALYSIS


1. When hearing a diversity case, a federal court will apply both state and federal law. To determine which law
to apply in which circumstance, the court conducts an Erie analysis. Dont be afraid of Erie. The entire
analysis is composed of only two steps.
2. Step 1: Do What Congress or the Constitution Says
a. Step 1 is pretty straightforward. The analysis is as follows: If a valid federal statute or constitutional
provision is on point, the federal court must apply the relevant provision. It is irrelevant whether the
provision is substantive or procedural, or whether it causes forum shopping, or otherwise seems
unwise to apply.
b. Validity
(1) Perhaps its obvious, but for a federal statute to be worthy of application by a federal court, the
statute must be valid. (The Constitution, of course, is per se valid.) Thus, if a federal statute
instructs federal courts to adjudicate cases one way for Caucasian plaintiffs and another way for
African American plaintiffs, the statute is invalid because it violates the equal protection
component of the Fifth Amendment. The federal court, when presented with such a statute in a
diversity case, should simply ignore it (i.e., pretend that it doesnt exist) and move to Step 2.
(2) The invalidity of a statute has special application in cases involving the Federal Rules of Civil
Procedure. Federal Rules are statutes themselves, but they are drafted by the judicial branch
and approved by Congress.
(a) What is invalidity?
To make sure that the judicial branch does not reach out and regulate the world outside
procedure, Congress ordained that a federal rule, to be valid, must not abridge, enlarge or
modify any substantive right.
(3) Thus, if a hypothetical rule declares that the defense of contributory negligence shall not be
valid in a federal court, and Congress for some reason approves it anyway, the rule is invalid
and should be ignored by the court. Importantly, this test is fairly easy to pass; no Rule has ever
been held to abridge, enlarge or modify any substantive right. So if you see a Federal Rule
that you are familiar with, it almost certainly is valid under this test and ought to be applied.
EXAMPLE: P sues D for defamation in violation of state law. The case was before a federal
court pursuant to diversity of citizenship. D argues that his statement, though defamatory, is
nonetheless protected by the First Amendment because P is a public figure. A federal court
should apply First Amendment law in deciding Jakes liability because such law is (obviously)
valid.
EXAMPLE: P sues D for negligence under state law. The case was brought before a federal
district court pursuant to diversity of citizenship. P sought to depose 15 witnesses, which is the
amount that state courts in that state would permit. Federal Rule of Civil Procedure 30, however,
limits parties to 10 depositions each. Because Rule 30 is valid (it is constitutional and does not
address substantive rights), the court should apply it and limit Ps depositions to 10.
c. On Point
(1) What does it mean to say that a statutory or constitutional provision is on point? This is
ultimately a question of interpretationwhether constitutional or statutory. If a valid Federal Rule
of Civil Procedure says, Lawsuits involving the rights of minors shall be sealed from public
view, then a lawsuit involving the right of a 13 year old should obviously be sealed from public
viewregardless of whether state law says that such suits should be open to public view.
(2) Not every question can be resolved so easily, however. Take, for example, a Supreme Court
case on this issue. P sued D in a diversity case. State law declared that tort suits must be
commenced within 2 years of the injury, and further specified that a suit is commenced when the
defendant is served with processnot when the complaint is filed. This was a problem for P,
who filed his suit before the 2-year mark but did not have the defendant served before the 2-
year mark. When D sought dismissal of the action, P sought refuge in the Erie doctrine. P
argued that Federal Rule of Civil Procedure 3, which says that a civil action is commenced by
filing a complaint with the court, was on point and thus controlled the matter, regardless of
what state law says. The Supreme Court disagreed, holding that, even though Rule 3 used the

12
term commence, the Rule does not pertain to when lawsuits are begun but rather how a
lawsuit is begun (i.e., by filing a complaint).
NOTE: It is impossible to lay out any rule for how to interpret Rule 3 or any other federal law.
What you need to know is simply that whether a provision is on point is an issue of
interpretation.
3. Step 2: If There Is No Federal Statutory or Constitutional Law on Point, Do What Would Avoid
Unfairness and Forum Shopping
a. The appropriate Step 2 analysis is as follows: If there is no valid federal statutory or constitutional law
on point, the federal court should follow its ordinary practices (whether rooted in case law, written
policy, informal practice, or simply habit) UNLESS doing so would lead to either a preference for one
court system over another or fundamental unfairness
b. Unfairness or Forum Shopping
(1) The easiest way to understand these concepts is by comparing two examples.
EXAMPLE: P sued D in federal court in a state law negligence action. As the trial is about to
start, P politely asks the federal judge to put on an old-fashioned, powdered wig. State law
requires state judges to wear such wigs, P explains, and because this is a diversity case, the
federal judge must follow state law unless a federal statute or constitutional provision instructs
otherwise. Since no federal statute or constitutional provision tells the judge how to dress for
trial, P argues, the judge must follow state law and put on the wig.
EXAMPLE: P sues D in federal court in a state law medical malpractice action. D moved to
dismiss because the plaintiff failed to present her grievance to a state board before filing suit.
State law, as part of an effort to deter spurious med mal claims, requires med mal plaintiffs to
first present their claim to a board of physicians and lawyers familiar with med mal law. Plaintiffs
who fail to take this step must, under state law, have their claims dismissed. Because P did not
take this step, D argues, her suit must be dismissed.
(2) If Step 1 represented the entirety of the Erie doctrine, the federal judge in these cases would
wear the wig in the first suit and dismiss the med mal claim in the second suit. After all, no
federal statute or constitutional provision instructs otherwise. But because of Step 2, the judge
would not wear the white wig but would dismiss the med mal case. The analysis is as follows:
EXAMPLE: Because there is no federal statutory or constitutional directive dictating the
appropriate attire for trial judges, the judge will stick to ordinary federal practice. By presiding
over the trial without a white wig, the judge will not make federal court materially more or less
attractive than state court. No rational litigant (i.e., a litigant focused on favorable results) would
care about that aspect of the proceedings.
EXAMPLE: Although there is no federal statute or constitutional directive ordering litigants to
first submit their claims to a board before filing them in federal court, the court should dismiss
Ps suit for failure to do so. Rational litigants would prefer to avoid that extra step and thus, if
they could avoid that step by filing in federal court, they would prefer federal court to state court.
To avoid this forum shopping, the federal court should thus follow state law.
(3) Courts and commentators have noted that, although the Supreme Court refers to forum
shopping and unfairness as two separate elements, unfairness is functionally subsumed into
forum shopping. That is, parties shop for forums that can give them an advantage; this
advantage is what the opposing party would refer to as unfair. Thus, a finding of forum
shopping will mean necessarily carry with it a finding of unfairness
c. What Happened to Substance and Procedure?
(1) You may remember the terms substance and procedure being common in the world of Erie.
Although courts still use these terms in Erie casesand the bar examiners may as wellthe
doctrine does not generally demand that a law be characterized in the first instance as
substantive or procedural. (A narrow exception is whether a federal rule modifies a substantive
right, but as noted above, no federal rule has ever been held to do this.)
(2) So dont fall into the trap of trying to figure out whether a particular law is truly substantive or
procedural. In the old days, courts tried to do this because the general rule was that federal
courts would apply federal procedure and state substantive law. But the Supreme Court
eventually realized that there was no such thing as true procedure or substance. So they
created Step 2 to figure out whether to apply a particular law or not. The Court never abandoned
the terms procedural and substantive, however; it simply applied those words to the result of
its Step 2 analysis. If Step 2 instructs the court to apply state law, the state law is called
substantive, regardless of its true nature.
(3) And if Step 2 instructs the court to stick with ordinary federal practice, that practice is called
procedural, regardless of its true nature.
4. Review of Steps 1 and 2
EXAMPLE: P sued D in Virginia federal court. D moved for summary judgment and attached several
deposition excerpts to his motionsomething that Federal Rule of Civil Procedure 56 allows him to do. P

13
objected, arguing that, under state law, summary judgment motions may not be supported by deposition
excerpts. By making summary judgment harder to obtain, this law implements Virginias important policy of
promoting jury trials. May D support his motion with deposition transcripts? Step 1: There is a federal
directive on pointnamely Federal Rule of Civil Procedure 56. The rule is valid because it does not
modify any substantive rights. Therefore, the court should permit the excerpts. Virginias stated policy
choices are irrelevant where Congress has made its own policy choices clear. Also irrelevant is the fact that
defendants may prefer federal court. Forum shopping is only a concern when a court engages in Step 2
analysis.
EXAMPLE: Although there is no explicit federal statute or constitutional provision addressing it, federal
courts have long prohibited attorneys from making per diem arguments during closing arguments. In a
per diem argument, the attorney asks the jury to consider the dollar amount of the plaintiffs pain and
suffering each day, and then multiply that amount by the number of days the plaintiff has or is expected to
suffer. Michigan law, however, explicitly allows this (something the plaintiffs bar lobbied hard for). Suppose
that P files a negligence case in federal court controlled by Michigan state law. May P make a per diem
argument in closing arguments? Step 1: There is no federal constitutional or statutory law on point, so go
to Step 2. Step 2: The federal court should follow Michigan law here as long as the per diem argument is
something that reasonable litigants would shop for or feel treated unfairly if denied.
5. Two Rules of Thumb
a. As you might imagine, Erie issues get litigated with some frequency. Because of this, there is general
agreement that certain types of state laws should be applied in federal diversity cases. Even if you
forget the two steps above, you may be able to get an Erie issue correct simply by remembering the
following.
(1) Statutes of Limitations Are Substantive
(a) When P sues D in a federal diversity action, Ps compliance with the statute of limitations
will normally be controlled by state law. That is, there is normally no federal law on point,
and the application of state law is necessary to prevent forum shopping/unfairness. Thus,
state statute of limitations law is applied (and labeled substantive).
(2) Choice of Law Rules Are Substantive
(a) Depending on the bar exam you are taking, you may or may not have much exposure to
choice of law rules. These rules instruct a court on the appropriate state law to apply when
a lawsuit touches on more than one state. In a federal diversity suit, Erie requires that
federal courts apply the choice of law rules of the state in which it is sitting. That is, there is
no federal law on point and conjuring up distinctly federal choice of law rules would only
encourage forum shopping. Thus, such rules are properly applied in diversity cases and
labeled substantive.
III. PRETRIAL PROCEDURES

A. THE COMPLAINT
1. Contents (Rule 8)
a. To initiate a lawsuit, the first step is filing a complaint. A complaint must contain three items.
(1) The grounds for subject matter jurisdiction
(a) For instance, if federal jurisdiction is based on diversity of citizenship, the complaint should
state that the plaintiff is a citizen of New York, the defendant is a citizen of California, and
the plaintiff seeks damages in excess of $75,000. If federal jurisdiction is based on the
presence of a federal question, the complaint should state, for example, that plaintiff
alleges that her claim arises under the Civil Rights Act and the Fourteenth Amendment to
the U.S. Constitution.
(2) A statement of facts
(a) The statement of facts should be sufficient to show that the plaintiff is entitled to relief. This
can often be done very simply, e.g., On May 1, 2013, the defendant neglected his duty to
yield to the plaintiff, and thus caused his car to impact the plaintiffs car, causing the plaintiff
significant damages.
(b) Although simple allegations like the above will often be sufficient, the Supreme Court has
warned that complaints must state facts that make the wrongdoing appear plausible, not
just hypothetically possible. A plaintiff cannot guess that the defendant has been negligent
and expect to make it past the pleading stage. Moreover, in cases alleging fraud or
mistake, the plaintiff must describe the alleged fraud or mistake with significant
particularity. General allegations will not suffice in these circumstances. We will discuss
plausibility a bit more when we discuss motions to dismiss later in the Bar Notes.
(c) Finally, a plaintiff is free to allege inconsistent claims. As long as each claim is, on it own,
supported by sufficient law and facts, it is properly plead.
(3) A demand for judgment and relief
(a) Finally, the complaint should ask for judgment and specify the relief sought.
14
2. Amendments (Rule 15)
a. Suppose a plaintiff files a complaint and then wants to change it? Can she do so? In some cases, she can
amend her complaint unilaterally and in others she must obtain permission. Either way, she may also have
to address potential statute of limitations issues.
b. The Ability to Amend
(1) As of Right:
(a) A party can unilaterally amend a complaint within 21 days of filing it, or, if an answer or
motion to dismiss has already been filed, within 21 days of that filing.
(b) A party can amend an answer within 21 days of filing it. (We will discuss motions to dismiss
shortly.)
(2) With Permission: If you cannot amend a pleading as of right, the only way to amend it is by
getting permission.
(a) Permission of opposing parties: If the opposing party or parties consent to the amendment,
a party may amend her pleading.
(b) Permission of the court: Regardless of whether the opposing parties consent, a party can
amend a pleading if the court permits her to. In considering whether to permit amendment,
the court will consider the reason for the delay in amendment and whether the amendment
will prejudice the opposing party
1) Permission to amend is usually liberally given by courts.
EXAMPLE: P sued D for breach of contract on May 1 and has D served the same
day. D files a motion to dismiss on May 15. Upon reading the motion to dismiss, P
realizes that she should have pleaded her claims differently. She has 21 days from
May 15 to automatically amend her complaint. P would have the same option if D had
filed an answer on May 15 rather than a motion to dismiss.
EXAMPLE: P sued D for breach of contract on May 1 and has D served the same
day. D filed an answer on May 15 but soon realized that he forgot to plead statute of
frauds as an affirmative defense. D can amend his answer within 21 days of May 15.
EXAMPLE: P sued D for breach of contract on May 1 and had D served the same
day. D filed an answer on May 15, and the case proceeded into discovery. During
discovery, P uncovered documents suggesting that D not only breached his contract
with P but defrauded her, as well. Because her 21 day window has passed, P can
only amend with permission of D or the court. The court is likely to grant P permission
as long as P only learned of the fraud during discovery and the amendment will not
significantly prejudice the defendant.
c. Amendments and Statutes of Limitations
(1) Because amendments, by definition, occur after a lawsuit has been filed, there sometimes arise
statute of limitations issues. For example, suppose P sues D on the day before the statute of
limitation expires, but then seeks to amend 5 days later. As noted above, she will have an
automatic right to amend her complaint, but that doesnt mean that her amended complaint will
comply with the statute of limitations.
(a) Whether an amendment has been filed within the statute of limitations depends on whether
the amendment seeks to add a new claim or a new party. (Amendments that do not add
new claims or parties, but merely correct errors or omissions in the complaint, will not
implicate statute of limitations concerns.)
(2) Amendments That Add New Claims: If a party amends a pleading by adding a new claim, that
amendment will be considered filed on the date on which the original complaint was filed as long
as the new claim arises from the same transaction or occurrence as the existing claim(s)
(a) This is often called the relation back rule, because it allows related claims to be
considered filed back at the time of the original suit.
EXAMPLE: P sues D on May 1, 2013 for damaging her car in a car accident. The statute
of limitations for this claim expired on June 1, 2013. On August 1, 2013, P began to
experience pain in her neck, and her doctor advised her that this was a latent injury from
the car accident. P thus amended her complaint to add a claim for physical harm. Ps
amendment will not be barred by the statute of limitations because it arises out of the same
transaction or occurrence as her original claim.
(3) Amendments That Add New Parties: Satisfying the statute of limitations when adding a new
party is much harder than when adding a new claim. Such amendments will only relate back to
the date of the original filing if:
(a) the party to be added knew of the lawsuit soon enough not to be prejudiced in defending it
and in no case later than the time permitted for service of process
(b) the party to be added should have expected to be named as a defendant, AND
(c) the party to be added was originally left out of the case because of a mistaken identity.
EXAMPLE: P sued Astro Inc., a large multinational company, for breach of contract. Astro

15
Inc. filed an answer truthfully denying that it had anything to do with the alleged contract.
During discovery, P realized that her contract was not with Astro Inc., but with Astro
Holdings LLC. Astro Holdings is a different legal entity but is managed by the same
persons who manage Astro Inc. P now wishes to amend her complaint to add a claim
against Astro Holdings (and dismiss her claim against Astro Inc.), but the statute of
limitations has run. Ps claim against Astro Holdings will probably be considered timely
because Astro Holdings knew of her suit within the SOL, knew it was the proper party, and
was most likely not sued because of Ps mistake in identifying it as the proper party.
EXAMPLE: P sued Hospital for injuries she suffered in a surgery. Her doctor learned of the
lawsuit when Hospital was served and was relieved not to be named as a defendant.
During discovery and after the statute of limitations has run, she learns that Hospital is in
horrible financial shape and will not be able to pay any judgment she obtains. She
therefore seeks to amend her complaint to add the surgeon who performed the operation.
This amendment will not relate back because, although the surgeon knew of the lawsuit
and may even have expected to be sued, P did not fail to sue him because of a mistaken
identity. She simply made a conscious decision not to sue him. Put differently, her mistake
was in litigation strategy, not in the identity of a would-be defendant.

B. SERVICE OF PROCESS (RULE 4)


1. Once a plaintiff has filed her complaint, the next step is to inform the defendant of the pending action. This
is called serving process.
2. What Is Process?
a. Process consists of two items: a copy of the complaint and a summons.
b. A summons is a court document that informs the defendant that the complaint has actually been filed
and that he must respond to it, or else be held in default.
c. Unless both the complaint and summons are served on the defendant at the same time, process has
not been completed.
3. When Must Process Be Served?
a. Process must be served within: 90 days of filing the complaint. PLEASE NOTE: This Is updated rule.
The deadline used to be 120 days.
b. If the plaintiff fails to have process served within that time, the court must dismiss the suit without
prejudice. However, If the plaintiff can show good cause why service was not completed within 90 days,
the court must provide the plaintiff additional time for service.
4. Who Can Serve?
a. Any person at least 18 years old and who is not a party to the action can serve process. Thus,
although plaintiffs are free to file their own complaints and argue cases pro se, they may not serve
process on their own.
5. How May Process Be Served?
a. The Federal Rules of Civil Procedure offer plaintiffs two options for serving process.
b. Option 1: Serve process in the manner prescribed by the state court in the state where the
federal suit has been filed OR by the state court in the state where the defendant will be
served.
EXAMPLE: P files a complaint against D in the Southern District of New York. D lives in New Hampshire and
will be served at home. If P desires, she can have D served in the manner prescribed by the New York
state courts (where the suit has been filed) or in the manner prescribed by the New Hampshire state
courts (where the defendant will be served).
(1) Of course, state rules will only be sufficient if they comply with constitutional minimums. If state
law permits service of process to be effectuated simply by flushing process down the toilet
without ever giving it to the defendant, then process will not be sufficient even though it complies
with state law.
c. Option 2: Serve process in the manner specified in Federal Rule of Civil Procedure 4.
(1) In the above action, if P is unfamiliar with service of process in the New York or New Hampshire
state courts, she is free to have process served in the manner specified in Rule 4. How service
works under Rule 4 depends on the nature of the defendant.
(2) Natural Persons
(a) When suing a natural person, the process server has 4 options:
1) deliver the process to the defendant himself wherever he may be;
2) leave process at the defendants usual place of abode with a person of suitable age
and discretion residing there
3) serve the defendants registered agent; or
4) mail process to the defendant with a letter requesting that the defendant waive
service in person.
a) If the defendant declines to waive in-person service, he is responsible for the

16
costs of personal service
(3) Corporations, Partnerships, or Associations
(a) To serve a corporation, partnership, or association, the process server has three options:
1) serve process on an officer;
2) serve process on a managing agent or general agent; or
3) serve process on any other agent authorized by appointment or by law to receive
service.
NOTE: If the agent is one authorized by law to receive service and the law requires
service by mail, service must be served by hand and by mail.
EXAMPLE: P sued D and hired a process server to serve him. The process server
walked up to D at a restaurant and handed him the summons and a copy of the
complaint. Service was lawfully accomplished.
EXAMPLE: P sued D and hired a process server to serve him. The process server
could not find D at work or on the street, so he went to Ds home. Ds 24-year-old
daughter who was visiting for the weekend answered the door and informed the
server that D was not present. The server left service with the daughter. Service was
not properly effectuated because the daughter, though somebody of suitable age and
discretion, did not reside at Ds home.
EXAMPLE: P sued D, a corporation that sells widgets. P saw Ds chief financial
officer at a cocktail party and served him process there. Process was not sufficient
because a party may not serve process. If someone other than P had served the
chief financial officer in that way, process would have been effectuated.
6. Relationship Between Service and Personal Jurisdiction
a. As we have seen, the law of PJ specifies the circumstances under which a federal court may
adjudicate the rights of a particular party. Just because a federal court has the authority to assert PJ
over a particular party does not mean that the court actually has PJ over that party. Some action must
be taken to activate the courts authority. That action is service of process. Thus, if a court has the
power to assert PJ over a party, that power becomes active at the moment the defendant is served
with process (or a waiver is filed with the court, if service is accomplished by a request for waiver).
NOTE: where a plaintiff relies on tag to subject the defendant to personal jurisdiction, the courts PJ is
both created and activated at the exact same moment.
7. Relationship Between Service and Constitutional Due Process
a. Due process is a phrase that can mean many, many things. At its core, however, the constitutional
right to due process entitles a defendant to notice of the claims against him and an opportunity to
respond to those claims. Service of process is one way that the notice requirement is satisfied, but it
is not the only way.
b. Notice through service
(1) If a defendant is served with process, the defendants constitutional right to notice has almost
certainly been observed. The key is whether service of process was reasonably calculated to
inform the defendant of the action against him. Most, if not all, service rules in the United States
meet this standard, but it is conceivable that state law could permit service in a constitutionally
deficient manner (e.g., service may be accomplished by handing process to the defendant
personally or by burying process in the desert on a starless night).
c. Notice without service
(1) Suppose that the defendant was not served with process at all, or served in a flawed manner.
Can the defendant raise a constitutional challenge to any subsequent lawsuit on these grounds?
Maybe. The key for constitutional purposes is not whether process was served, but rather,
whether the plaintiff took steps that were reasonably calculated to inform the defendant of the
action against him.
EXAMPLE: P lives next door to an abandoned house that is overrun with wild animals and is
thus a nuisance under state law. The local property records state that the owner of the house
lives in a different state. P files suit in federal court against the owner and attempts to have him
served with process. All methods of service fail, however. P can still litigate her suit against the
owner as long as she makes every reasonable effort to inform the owner of the suit, such as,
perhaps, hiring a private investigator to find the owner or publishing notice of the lawsuit in
appropriate newspapers.
EXAM TIP: Key test for notice Its not whether process was served, but rather whether the
plaintiff took steps that were reasonably calculated to inform the defendant of the action against
him
8. New Claims Filed During the Lawsuit
a. Often, a lawsuit will involve many more claims than the plaintiffs original claim. Not only can the
plaintiff add claims along the way, the defendant can counterclaim against the plaintiff, cross claim
against a co-defendant, or implead another party to share the liability.

17
b. The question arises: Must process be served every time a claim is filed? In short, no. Service of
process need only be accomplished when a new party has been added to the lawsuit (such as
through impleader, which we will get to shortly). Without process on the new party, the courts power
of PJ over the party will not be activated. For counterclaims, cross claims, and other claims against
current parties, however, service of process is unnecessary.

C. RESPONDING TO A COMPLAINT (RULE 12)


1. After process is served, the party sued must decide how to respond. The Federal Rules offer several
options:
a. Answer
(1) An answer is a pleading in which the defendant admits or denies the plaintiffs allegations and
lists any defenses he might have. Sometimes a party may wish to raise defenses before filing an
answer by filing a motion to dismiss. We will discuss motions to dismiss in the section on Pretrial
Adjudication.
(2) Admissions or Denials
(a) For each allegation in the plaintiffs complaint, the defendant should specifically admit or
deny the allegation.
(b) If the defendant fails to deny an allegation, it is deemed admitted.
(3) Defenses
(a) Available Defenses: The defendant should also raise any defenses he has to the plaintiffs
claims. There is no limit to the available defenses, but common ones include: statute of
limitations, statute of frauds, and assumption of risk.
(b) Waiver: If the defendant fails to include a defense in his answer, he will, in most cases,
forfeit that defense. Defenses that are not forfeited include: failure to state a claim upon
which relief can be granted, failure to join a necessary party, and lack of subject matter
jurisdiction. (We will return to waiver again later)
b. Motion for More Definite Statement
(1) If a complaint or answer is so vague or ambiguous that the responding party cannot reasonably
prepare a response, the responding party may move the court to order the pleading clarified.
c. Motion to Strike
(1) If a complaint or answer contains redundant, immaterial, or scandalous matter, the court may
on its own or by motionorder the improper material stricken.
d. Motion to Dismiss
(1) We will discuss motions to dismiss more fully below, but for now, simply know that a motion to
dismiss seeks the dismissal of a claim and can be filed by any party defending a claim.
2. Timing of Responses
a. Initial Response
(1) Process Actually Served: If D was actually served with process (regardless of the method), D
has 21 days within which to respond.
(2) Service of Process Waived: If D waived service of process, D has 60 days to respond.
b. Secondary Response
(1) Suppose that D responds to Ps complaint by filing a motion for more definite statement, a
motion to strike, or a motion to dismiss. What must D do if the motion is denied? D must file an
answer within 14 days of the denial.
EXAMPLE: P sues D on May 1 and asks him on the same day to waive service of process. D
agrees and files a motion to dismiss on June 1, which is denied on July 2. D then files an
answer on July 17. D timely filed his motion to dismiss (he had 60 days to file it) but did not
timely file his answer (he had 14 days to file it).

D. ETHICAL OBLIGATIONS (RULE 11)


1. Contrary to public opinion, honesty and decency really do matter in litigation. In federal court, Federal Rule
of Civil Procedure 11 addresses these issues.
2. Standard of Conduct
a. By presenting a document to the court (or later orally advocating the information in the document), an
attorney or unrepresented party certifies that, after making a reasonable inquiry, its submission is
based on:
(1) Good Faith
(a) The party certifies that the submission is not for any improper purpose such as to harass,
delay, or needlessly increase the cost of litigation.
(2) Good Facts
(a) The party certifies that the submission is based on evidentiary support or, if specifically
noted, will likely have evidentiary support after discovery. This applies to affirmative

18
representations of fact, as well as denials of factual allegations.
(3) Good Law
(a) The party certifies that the submission is warranted by existing law or presents a non-
frivolous argument for a change in existing law.
3. Sanctions
a. How sought
(1) To seek sanctions for a violation of Rule 11, the complaining party must first draft a motion for
sanctions and serve it on the offending party.
(2) The motion should not be filed, however, until 21 days after service on the offending party. The
21-day period is a safe harbor for the offending party; it allows him a chance to withdraw or
modify any submission allegedly in violation of Rule 11.
b. Nature of sanctions
(1) A court may order monetary or non-monetary sanctions. The goal of the sanctions must be to
deter repetition of such misconduct by others in a similar position
c. Who is subject to sanctions?
(1) Any person or entity that has committed a Rule 11 violation or is responsible for a Rule 11
violation is subject to sanctions, except that parties may not be subject to monetary sanctions
for violating the good law requirement.
(2) When an attorney is found to have violated Rule 11, the attorneys law firm must be held jointly
liable absent exceptional circumstances.
4. Not Applicable to Discovery
a. The obligations imposed by Rule 11 do not apply to any discovery matter, whether it is a discovery
request, response, motion, or any other matter. Misconduct in the realm of discovery is addressed by
discovery-specific rules (a topic we will come to shortly).
5. Review of Rule 11
EXAMPLE: P contacted her attorney and sought assistance filing a lawsuit arising from a car accident.
Although P claimed that the accident happened last week and that she suffered serious injuries, the
attorney filed a complaint on her behalf without verifying that she was actually injured. During discovery, it
became clear that P was never injured and had made up the entire story. P is subject to sanctions because
she is (at least partially) responsible for the Rule 11 violation. The attorney is subject to sanctions because
he did not take reasonable steps to investigate the plaintiffs claims. If the attorney works for a law firm, the
firm should be found jointly liable for the attorneys sanctions.
EXAMPLE: In 1951, Thurgood Marshall filed suit challenging the segregation of school children based on
their race. At the time, segregation was lawful under Plessy v. Ferguson. Marshalls actions would not have
violated Rule 11 because he was advocating in good faith for a change in existing law.
EXAMPLE: P was arrested several years ago for public drunkenness. D found this out and posted it on a
website where many ridiculed P. P sued D for defamation. P has likely violated Rule 11 because his claim
is not supported by good law; truth is a complete defense to defamation. If P made it clear in his complaint
that he was seeking a change in existing law and offered a reasoned justification for that change, P would
likely not be in violation of Rule 11.
EXAMPLE: P sued D for medical malpractice. During discovery, P sought a copy of Ds medical license. D
forged a license and submitted it to P. D has not violated Rule 11 because Rule 11 does not apply to
discovery matters. D will still be subject to sanctions, however, based on different rules.

E. PROVISIONAL RELIEF (RULE 65)


1. In General
a. Sometimes a plaintiff files suit seeking injunctive relief, but cant litigate the suit fast enough to get
that relief before the defendant commits some irreversible harm.
EXAMPLE: If Developer is intent on tearing down a historic building, Ps suit to enjoin Developer may
ultimately be worthless if Developer can continue with the teardown during the pendency of the
litigation. In this situation, P should ask for a preliminary injunction or a temporary restraining order.
2. Preliminary Injunction
a. A preliminary injunction (PI) is available only upon notice to the adverse party, and satisfaction of the
following elements:
(1) Likelihood of Success on the Merits
(a) Ps legal claim is not only plausible, but if she litigates it to conclusion, she will likely win.
This means that the law and facts relevant to the case are both on the plaintiffs side.
(2) Irreparable Harm
(a) If P is not granted a preliminary injunction, she will likely suffer harm that cannot be undone
by later relief. In practice, this requirement is easier to satisfy than might be imagined. Courts
typically find that any violation of a right counts as irreparable, even if monetary relief would
be available after the fact.
(3) Balance of Hardships

19
(a) The court will consider whether the harm to P if the injunction is denied is greater than the
harm to D if the injunction is granted
EXAMPLE: If P seeks a PI barring D from tearing down a historic property to build a gas
station, the court will balance the harm to P (loss of a historic property forever) against the
harm to D (delay in construction for a period of months). In this circumstance, P would
likely suffer the greater harm.
(4) Public Interest
(a) The court will not issue a PI that is injurious to the public interest.
EXAMPLE: If P seeks an injunction against Ds sale of a flu vaccine, the recipe for which D
allegedly stole from P, the court would likely be reluctant to issue a PI if D was the only
supplier of the vaccine to the public.
(5) Payment of Security
(a) Suppose a plaintiff obtains a PI but does not prevail in the suit? The defendant will have
had his work delayed and perhaps suffered losses because of it. For this reason, a plaintiff
seeking a PI must deposit enough money with the court to compensate the defendant for
any losses it might suffer if the suit is ultimately unsuccessful.
3. Temporary Restraining Order
a. Similar to a PI
(1) A temporary restraining order (TRO) is very similar to a PI. In deciding whether to grant a TRO,
the court applies the same five-part test (above) as applied in PI requests.
b. But different, too. There are two key differences between a PI and TRO.
(1) Timing: Unlike a PI, which remains in effect during the pendency of a suit, a TRO expires 14
days after issuance. The TRO can be extended for an additional 14 days upon good cause
shown.
(2) Notice: Unlike a PI, which can be granted only in an adversary proceeding, a TRO can be
granted ex parte.
(a) To obtain an ex parte TRO, however, the applicant must show the applicant tried to notify
the opposing party, was unable to, and needs a TRO to avoid immediate irreparable harm.
F. JOINDER
1. The law of joinder is about the size of a lawsuit. The minimum size of a lawsuit is one plaintiff versus one
defendant involving one claim. But what is the maximum size? The sky is the limit, subject to the following
rules.
2. Joinder of Claims
a. Multiple claims brought by a single claimant and single defendant (Rule 18)
(1) If a plaintiff has numerous claims against a single defendant, she is free to bring all of those
claims in the same lawsuit regardless of whether they are related to each other
(2) The law of joinder does not require a plaintiff to bring every claim she has against a particular
defendant; it simply permits her to do so. Nonetheless, if the plaintiff refuses to bring a claim that
is related to the other claims in her suit, the law of preclusion (which we will address later) will
often bar litigation of the claim the plaintiff omitted.
EXAMPLE: P purchased a summer pass from D, an amusement park. During one visit, the park
refused to honor her pass and she was forced to purchase entry at the gate for $100. Several
weeks later, she presented her pass and was properly admitted, only to later injure herself on a
ride. If P wishes to sue D for both injuries (being denied entry and being harmed on a ride), she
may do so but is not required to do so. If she decides to file only one claim, she can litigate the
other claim later because the claims are not related to each other.
b. Counterclaims (Rule 13(a), (b))
(1) Defined
(a) A counterclaim is how a party being sued returns fire. Usually the party being sued is a
defendant and is returning fire against the plaintiff, but any party being sued can return fire
with a counterclaim. Thus, if a defendant files a counterclaim at the plaintiff, the plaintiff
may fire a claim back at the defendant, thus filing a counterclaim to the counterclaim.
Similarly, if a defendant files a cross claim or impleader claim (both discussed shortly), the
parties defending themselves against those claims can return fire against the defendant.
(2) Permissive and Compulsory Counterclaims
(a) A permissive counterclaim (see Rule 13(b)) is a claim that does not arise out of the
transaction or occurrence underlying any of Ps claims against D. A defendant may file a
permissive counterclaim but is not required to do so.
(b) A compulsory counterclaim is a claim that does arise out of the transaction or occurrence
underlying any of Ps claims against D.
1) If a defendant has a compulsory counterclaim, he must file it or else he will forfeit
the claim in any future litigation
(3) Multiple Counterclaims
20
(a)If a party has multiple counterclaims, he may bring them all together regardless of whether
they are related.
(b) Any claim that is related to the plaintiffs initial volley is compulsory and must be brought or
else forfeited.
c. Cross Claims (Rule 13(g))
(1) Defined: A cross claim is a claim filed by a party against a co-partyi.e., someone on the same
side of the v. Cross claims may be filed only if they arise from the same transaction or
occurrence underlying one of the plaintiffs claims
(2) Claims Joined With Cross Claims: Once a defendant files an appropriate cross claim against a
co-defendant (i.e., a claim arising out of the same transaction or occurrence underlying Ps suit),
the defendant may join with that claim additional claims, even if those claims are unrelated to
the transaction or occurrence underlying Ps suit. This is simply an application of the rule,
discussed above, that allows plaintiffs to file multiple claims, even if the claims are not related to
each other.
(3) Returning Fire to a Cross Claim: Once a defendant files a cross claim against a co-defendant,
that co-defendant is free to respond with a counterclaim. Like an ordinary counterclaim, a
counterclaim filed in response to a cross claim can be compulsory or permissive.
EXAMPLE: P is injured in a bus accident when the bus driver fell asleep at the wheel. P sues
the bus company and the driver. The bus company files a claim against the driver for
contribution, arguing that its potential liability is solely the result of the drivers negligence. The
driver then files a claim against the bus company, alleging that the company has wrongfully
withheld his overtime pay over the past year. Both claims are permissible under the law of
joinder. The bus companys claim is a cross claim and is permissible because it relates to the
transaction or occurrence underlying the plaintiffs claim. The bus drivers claim is a permissive
counterclaim, which may (but is not required to) be brought. Note that the bus driver could not
file his claim until the company opened the door by filing a cross claim.
d. Impleader Claims (Rule 14)
(1) Defined: Like a counterclaim or cross claim, an impleader claim is a claim brought by a party
defending a claim. Unlike those claims, however, an impleader claim is brought against
somebody not yet involved in the lawsuit and may only be brought if the claim alleges that the
third party is responsible for some or all of the liability facing the defending party.
NOTE: The classic impleader claims are contribution claims (claims against a joint tortfeasor)
and indemnity claims (claims against an insurer).
EXAMPLE: P is injured in a bus accident when the bus driver fell asleep at the wheel. P sues
the bus company but, unlike the example above, does not sue the bus driver. The bus company
then files two claims: one against the driver for contribution, (arguing that its potential liability is
the result of the drivers negligence) and a second against its insurance company (arguing that
the company has a duty to pay any judgment issued against the bus company). Both claims are
permissible under the law of joinder.
(2) Timing: A defendant has an automatic right to implead a third party defendant if he does so
within 14 days of serving an answer. If 14 days have already elapsed, a defendant may only
implead a third party defendant with the permission of the court.
(3) Subsequent Claims: After a party has been impleaded into a lawsuit, the party may bring claims
of its own against other parties or implead additional parties. Additionally, the original plaintiff in
the suit may file a claim against the impleaded defendant provided the claim relates to one of
the plaintiffs original claims.
EXAMPLE: After the bus company impleaded the bus driver, the plaintiff filed a claim against the
driver for causing her injuries by falling asleep at the wheel. The plaintiffs claim is permissible
under the law of joinder.
3. Joinder of Parties
a. Permissive Joinder (Rule 20)
(1) Under the federal rules, multiple plaintiffs can join together in a single action, or multiple
defendants can be sued in a single action, as long as:
(a) the joined parties claim relief (if plaintiffs) or face liability (if defendants) that arises out of
the same transaction or occurrence, and
(b) there will arise in the action questions of law or fact that are common to the joined parties
EXAMPLE: P1 and P2 were passengers on a bus when they were injured in a crash they
believe was caused by the negligence of the bus driver. They may join together to sue D1, the
bus company, and D2, the bus driver, for their injuries. P1 and P2 can join together as plaintiffs
because their claims arise from the same transaction or occurrence (the crash), and there will
be questions of law or fact relevant to both of their claims (the negligence of the driver). D1 and
D2 can be sued jointly because their potential liability arises from the same transaction or
occurrence (the crash), and there will questions of law or fact common to the liability of the
21
driver and bus company (the negligence of the bus driver).
b. Mandatory Joinder (Rule 19)
(1) In contrast to the above, there are occasions where plaintiffs may be forced to add a party to a
lawsuit. Where deciding a lawsuit would significantly impact a person not part of the lawsuit, the
federal courts are reluctant to proceed. To determine whether a missing person is important to a
suit and whether to litigate the suit in that persons absence, federal courts work through the
following three steps. Note that mandatory joinder motions are typically made by defendants in
motions to dismiss pursuant to Rule 12(b)(7).
(2) Step 1: Determine whether the absent party is necessary.
(a) A party is necessary if the party has an interest that might be impaired if the party is left
out of the litigation, if complete relief cannot be issued in the partys absence, or if the
current parties would be subject to duplicative or inconsistent liability.
(b) If a party is not necessary mandatory joinder does not apply and further steps can be
skipped
EXAMPLE: P leased a valuable painting to D. P believed the lease term was 3 years, and
D believed it was 4. After possessing the painting for 2.5 years, D leased the painting to X
for 1 year. When the 3-year mark arrived, P brought suit to reclaim the painting. X is a
necessary party because it has an interest in the subject matter of the litigation (the
painting), because D could be subject to duplicative liability (D could end up being liable to
both P and X) and because the court cannot order the painting returned unless the person
with the painting (X) is within the jurisdiction of the court.
EXAMPLE: P was struck by a taxi cab while crossing the street. P sued D, the taxi cab
company. D argues that X, the driver of the cab, is a necessary party. This argument will
fail. As a rule, joint tortfeasors are not necessary parties. There is no mandatory joinder in
this situation.
(3) Step 2: If the missing party is necessary, determine whether she can be joined. If the missing
party can be joined, the court should join the party and simply adjudicate the case (thus skipping
Step 3). If the missing party cannot be joined, the court should proceed to Step 3. There are two
reasons why a missing party cannot be joined in federal court:
(a) the court lacks personal jurisdiction over the missing party; and
(b) adding the party would destroy federal jurisdiction based on diversity of citizenship
EXAMPLE: P sues D in Ohio federal court to reclaim a painting she leased to D. D leased
the painting to X, rendering X a necessary party. X is a Michigan resident and has never
been to Ohio or had any contact with the state. The court may not join X, even though he is
a necessary party, because he is not subject to personal jurisdiction in Ohio.
EXAMPLE: P sues D in Ohio federal court to reclaim a painting she leased to D. P is a
citizen of Michigan, and D is citizen of Ohio. D leased the painting to X, rendering X a
necessary party. X is a Michigan citizen. The court may not join X, even though he is a
necessary party, because X would be joined as a defendant and thus diversity would be
destroyed.
(4) Step 3: If the necessary party cannot be joined, determine whether the necessary party is
indispensable.
(a) If a missing party is indispensable, the court must dismiss the suit; if not, the court may
adjudicate the case in the partys absence.
(b) To determine whether a missing party is indispensable, the court will consider: the extent of
prejudice to the missing party; whether such prejudice can be lessened by shaping the
relief in a particular way; and whether the plaintiff, if the case is dismissed, will be able to
find relief in another forum. The courts overarching goal is to see that the dispute be
resolved with as little prejudice as possible to the missing party.
EXAMPLE: P sues D in Ohio state court to reclaim a painting she leased to D. X, a necessary
party, resides in Michigan and is not subject to personal jurisdiction in Ohio. D is subject to
personal jurisdiction in Michigan. Given this, a federal court might dismiss the suit because P
could obtain relief by re-filing the entire suit in Michigan. Keep in mind that these decisions are
highly discretionary.
c. Class Actions (Rule 23)
(1) A class action is a form of litigation where one person (the representative) litigates a dispute on
behalf of a group (the class). This form of litigation is permissible if: (1) a class can be formed,
and (2) the action brought is of the sort proper for resolution via a class action.
(2) Four Requirements: To form a class for the purposes of litigation, the following four
requirements must be met. Further, as illustrated in the examples following these requirements,
the stated description of the class is important.
(a) Numerosity The class is so numerous that joinder of all members is impracticable
(b) Commonality There are questions of law or fact common to the class

22
(c) Typicality: The claims of the representative parties are typical of those of the class, thus
ensuring the representative will have incentives to litigate in ways that will protect the class
(d) Representativeness: The representative parties will fairly and adequately protect the
interests of the class. The requirement of representativeness overlaps with typicality, but it
spreads more broadly to any circumstance (such as an inexperienced attorney) that might
adversely impact the interests of the class.
EXAMPLE: P bought a set of bed sheets from D, a large national dry goods store. D
advertised the sheets as 1000 Thread Count, but in fact the sheets only had a thread
count of 500. D has been selling these falsely advertised sheets for several years. P brings
suit, and her attorney seeks to have a class certified containing all persons who
purchased those same sheets. A court will likely certify the class because: (1) presumably
a large number of other persons were injured, (2) the members of the class all have claims
with a common issue of law or fact (buying a product that was mislabeled), (3) Ps claim is
apparently typical of all the other claims, and (4) on these facts, there do not appear to be
any problems with the capacity of the plaintiff and her attorney to represent the interests of
the class.
EXAMPLE: P became nauseated after ingesting some heartburn medication and sued D,
the manufacturer of the medication. P asked the court to certify a class containing all
persons injured by the medication. A court will not likely certify this class because there
are major problems with commonality and typicality. All persons injured would include
users of the medication that suffered excessive sweating because they ingested the drug
in large doses, those who suffered vertigo because they were using additional medications
at the same time, etc. Certification might be permissible if the class was defined instead as
all persons who suffered nausea within 2 hours of ingesting the drug in prescribed
amounts. This would cut down on numerosity, but assuming the problem is sufficiently
widespread, there would still be enough potential plaintiffs to form a class.
(3) If a class can be formed, is the suit nonetheless proper for adjudication as a class
action? Class action adjudication is proper in any of the three instances below.
(a) Where separate actions would create a risk of inconsistent judgments or judgments that
would substantially impair the ability of a nonparty member to protect his interests. (Note
how the concerns underlying this type of class action are similar to those underlying
mandatory party joinder, discussed just above.)
EXAMPLE: The city of Milwaukee, Wisconsin plans to build a bridge in Area A. The citizens
of that area think the city is obliged by federal law to build it in Area B and have sought an
injunction barring its construction. For their part, the citizens of Area B believe that the bridge
must be built in Area A because building it in Area B would violate federal law. This issue may
be resolved through class litigation so that the city can know for sure where to build the bridge.
(b) Where the party opposing the class has acted in ways generally applicable to the class.
(Note that class adjudication under this option is unlikely, but not impossible, if the suit
seeks monetary relief.)
EXAMPLE: Inmates in the California prison system believe that they are constitutionally
entitled to food that is healthy and desire an injunction ordering the state to provide such
food. The inmates may litigate their claim as a class because the state is treating the entire
class in the same way. If the class members each sought damages, class adjudication
would most likely not be appropriate because the states misbehavior has not likely
impacted each inmate the same way.
(c) Where the court finds that common questions of law or fact predominate over
individualized questions and that a class action is superior to other available methods for
the fair and efficient adjudication of the controversy.
EXAMPLE: Company X is a publicly traded company. Recently, its share price dropped 20%
after it surfaced that sales figures had been fraudulently inflated over the past two years. The
owners of shares in Company X lost significant amounts of money and would like to sue as a
class. The shareholders can sue as a class because the question of Company Xs fraud is a
common question to all members of the class and litigating the matter as a class is far
preferable to litigating thousands of individual suits. Note the damages awarded to each
class member may be different (depending on the amount of shares and time of purchase),
but such individualized questions are easy to resolve and do not predominate over the
common question of fraud.
(4) Additional Considerations
(a) Personal Jurisdiction: In a class action case, the court must have personal jurisdiction over every
defendant and only the named plaintiffs. (PJ over the named plaintiffs is easy because they
consent to jurisdiction by filing the suit.) The court does not need personal jurisdiction over each
and every class member.

23
(b) Binding Effect: A judgment in a class action will bind all members of the class unless they
opt out of the class.
1) Class members have the option of opting out in so-called common-question class
actionsi.e., class actions where common questions of law or fact predominate over
individualized questions. (See Company X example above.)
(c) Notice: In common-question class actions, class members must be given notice of the
pendency of the suit. The notice must be reasonably calculated to apprise members of the
class, and is often accomplished through mail or publication.
(d) Subject Matter Jurisdiction: When a class action alleges a violation of state law, it may
appear in federal court only if the defendants and the representative plaintiffs (not all class
members) are completely diverse from each other.
1) However, if the class has over 100 members and seeks damages over $5,000,000, a
federal court may assert jurisdiction over the suit if any single class member is
diverse from any single defendant.
2) Jurisdiction can be established through an initial filing in federal court or through
removal. Moreover, unlike ordinary removal rules, removal can be made by a home
state defendant and can be made w/o agreement of all other Ds.
EXAMPLE: P is the lead plaintiff in a class action composed of 5,000 class members
from all 50 states. The defendants are citizens of Delaware, New York, Illinois, and
California. The amount in controversy is $6,000,000. The plaintiff may file suit in
federal court because at least one of the class members is diverse from at least one
of the defendants, and the amount in controversy is over $5,000,000. If the plaintiff
elects to file in state court, any single defendant can remove the case to federal court
(even if the other defendants dont join in removal, and even if the defendant has
been sued in his home state).
d. Interpleader Claims (Rule 22)
(1) Defined: A party may file an interpleader claim when the party fears that it will be caught in the
middle. That is, it fears that it will be exposed to multiple and inconsistent liability
EXAMPLE: X died leaving behind both an ex-wife and a widow. Each believed she was the one
entitled to Xs life insurance proceeds. Life Insurance Co., who wrote the policy, may file an
interpleader action because it is caught in the middle. If it pays the proceeds to ex-wife, widow
will sue it. If it pays the proceeds to widow, ex-wife will sue it.
(a) Note that if a party decides not to sue a single party instead of multiple parties using an
interpleader claim, the party left out be may be required to join as a necessary and
indispensable party (discussed above) or may elect to intervene in the suit on her own
(discussed immediately below).
e. Intervention (Rule 24)
(1) Sometimes a non-party is interested in the result of a lawsuit but has not been joined as a co-
party (through permissible or mandatory joinder) or joined as a third party defendant (through
impleader). In these circumstances, the non-party may attempt to join the lawsuit through her
own initiative.
(2) Intervention of Right: Upon timely application, any person must be permitted to intervene in an
action when the applicant:
(a) claims an interest relating to the subject matter of the action, AND
(b) is so situated that, absent the intervention there is a risk that the applicant will not be able
to protect that interest.
EXAMPLE: P sues D for the return of a painting P had leased to D. The painting is now in
the possession of X, who desires to keep it and believes she has the right to do so. P did
not sue X in her original action against D because the court would not have PJ over X. D
declines to move to dismiss for mandatory joinder, hoping to keep X out of the lawsuit.
Nonetheless, X may intervene as of right because she has an interest related to the
subject matter of the suit and may not be able to protect that interest unless she is involved
in the suit. Note that there are no PJ problems because, by electing to intervene, X has
consented to the courts authority over it.
(3) Permissive Intervention: Upon timely application and at the courts discretion, any person may
be permitted to intervene in an action when the intervenor has a claim or defense that shares
with the main action a common question of law or fact.
EXAMPLE: D is a factory that emits large amounts of smog on a daily basis. P1, who lives near
Ds factory, sued D for common law nuisance. P2, another person who lives in the area and
suffers from the smog, read about the lawsuit in the paper and would like to intervene. P2 is
eligible to join in the lawsuit because it has a claim or defense that is based on facts or law in
the main action. Note that the court has broad discretion to permit or deny intervention.
4. The Relationship Between Joinder and SMJ, PJ, and Venue

24
a. The law explained above is the law of joinder. The law of joinder, however, does not displace some of
the other law we have discussed. What happens, for example, if the law of joinder allows a claim to
be added but the additional claim is not consistent with the law of SMJ, PJ, or venue?
b. Subject Matter Jurisdiction
(1) SMJ is a necessity for every claim, whether it is a counterclaim, cross claim, impleader claim, or
any other claim in a lawsuit. Thus, even if a claim can be joined, the claim cannot actually be
litigated in federal court unless it can be heard under the courts federal question, diversity, or
supplemental jurisdiction.
EXAMPLE: P sues D, a non-diverse defendant, for a violation of federal anti-discrimination law.
D files a counterclaim against P under state law for damaging company property while she was
employed there. Ds claim is permissible under the law of joinder but cannot be heard in federal
court because there is no SMJ. The claim does not present a federal question, is not against a
diverse party, and is not related to a claim falling within the courts original jurisdiction.
c. Personal Jurisdiction
(1) PJ is a necessity for every party. (Remember, in class actions, class members are not parties
for the purposes of personal jurisdiction; only the lead plaintiffs are parties.) Thus, if a plaintiff
sues multiple defendants, or a defendant impleads a third party, the court must ascertain
whether each and every party before it is subject to the power of the court.
(a) To do so, the court will apply the rules of personal jurisdiction we learned above. Recall
those rules briefly. Step 1: Look at state law for the state in which the federal court sits.
Step 2: Determine whether the assertion of personal jurisdiction authorized by state law is
constitutional.
(2) Although this works for almost all instances where joinder brings a new party into a suit, there is
one instance where an additional option is available. When we discussed Step 1 back in the
Personal Jurisdiction section, we noted that there are a couple rare instances in which Congress
has authorized federal courts to assert personal jurisdiction over a defendant without regard to
state law. Those situations are few, but one of them applies to impleaded defendants. The rule is
as follows:
(a) Bulge Rule: An impleaded defendant will be subject to the personal jurisdiction of a
federal court if, in addition to any other ground for personal jurisdiction, the impleaded
defendant is served with process within 100 miles of the courthouse where the suit is
pending.
EXAMPLE: P was driving in her home state of Mississippi when D, a resident of Florida
traveling in Mississippi, injured her. She sued D in Mississippi federal court for $100,000. D
believes that the accident was the result of faulty repairs made to the car back in Florida by
his local mechanic. D wishes to implead the mechanic to share in the liability. Although
joinder of the mechanic is permissible, and the court has SMJ over the impleader claim,
the court may not adjudicate the claim because it has no PJ over the mechanic.
EXAMPLE: P sued D1 in the Eastern District of Missouri, filing suit in the St. Louis federal
courthouse. D1 believed that D2 ought to share in its potential liability and so impleaded
D2. D2 lives in East St. Louis, which is the in the state of Illinois. D2 may not be subject to
personal jurisdiction under the rules we learned above, but D2 may nonetheless be forced
to stand trial in Missouri if he is served in East St. Louis (or anywhere else within 100 miles
of the courthouse).
d. Venue
(1) Unlike SMJ and PJ, the law of venue need only be consulted when the lawsuit is filed. If claims
are joined after the initial filing, those claims may not be challenged for being brought in an
improper venue.
EXAMPLE: P lives in Kansas and D lives in Illinois. They took a road trip together to Maine
where D drove the car off the road, causing P $100,000 in damages. P sued D in his home state
of Illinois. After being sued, D counterclaimed for $100,000 in injuries he suffered last year while
visiting P in her home state of Kansas. The court can hear Ds counterclaim even though, had D
filed the claim as a plaintiff, he could not lay venue in Illinois.

G. PRETRIAL CONFERENCES AND SCHEDULING ORDERS


1. Rule 26(f) Conference (a.k.a. Discovery Planning Conference)
a. Rule 26(f) requires parties to meet and discuss the likely content of discovery in the case and draft a
discovery plan.
b. The plan must be submitted to the court within 14 days of the conference.
c. The Rule 26(f) conference must be held at least 21 days before a Rule 16(b) conference is held or
order is issued.
2. Rule 16(b) Conference and Order
a. After a Rule 26(f) conference has been held, the court may order the parties to confer again to

25
discuss the litigation and, in particular, the most efficient way for it to proceed and any possibilities for
settlement.
b. Regardless of whether the court orders this conference, Rule 16(b) requires the court to issue a
scheduling order. The scheduling order will dictate the schedule upon which the litigation will progress
(e.g., periods within which parties can be added, motions made, etc.) and specify particular rules
pertaining to discovery (e.g., the scope of discovery on particular issues).
(1) The order must be issued within 90 days of when the defendant is served, or within 60 days of
when the defendant appears, whichever is earlier. PLEASE NOTE: This Is updated rule.
Deadline used to be within 120 days of when defendant served or within 90 days of appearance.

H. DISCOVERY
1. Discovery is the portion of a lawsuit in which the parties can learn information from each other and from
non-parties. What you need to know about discovery can be divided into four separate topics: (1)
mandatory disclosures, (2) scope, (3) methods, and (4) enforcement.

2. Mandatory Disclosures (a.k.a. What You Will Get Without Asking) (Rule 26(a))
a. Most discovery operates through specific requests made by a party. When a proper request is made,
the party receiving the request has the obligation to provide the information sought. In some
instances, however, a party must provide information to the opposing party even when a request has
not been made. There are three types of these mandatory disclosures: initial, expert, and pretrial.
b. Initial Disclosures
(1) At the outset of a case, each party must provide to all other parties certain information.
(2) Timing: These disclosures must be made within 14 days after a Rule 26 conference.
(3) Witnesses: Each party must provide to all other parties the name and contact information of any
witness that a party may use to support a claim or defense. If the witness will only be used for
the purposes of impeachment, the party need not disclose that witness.
(4) Documents or Objects: Each party must provide to all other parties copies of (or descriptions of)
documents, electronically stored information (ESI), and tangible objects that a party may use to
support a claim or defense. PLEASE NOTE: This is an updated rule on ESI.
(5) Damages: Each party must provide to all other parties a computation of damages the party
seeks, as well as any documents or other materials supporting that computation.
(6) Insurance Agreements: Each party must provide to all other parties copies of an insurance
agreement that may require an insurance company to pay all or part of a judgment.

c. Expert Disclosures
(1) At least 90 days before trial, each party who plans to rely on expert testimony must disclose to
all other parties the name and contact information of the expert, as well as the experts final
report.
(2) The report must explain the experts qualifications, her opinion and the information on which she
relied
d. Pretrial Disclosures
(1) At least 30 days before trial, each party must provide to all other parties:
(a) a list of the witnesses she expects to call at trial,
(b) the witnesses she may call if the need arises,
(c) a list of witnesses whose testimony will be presented through a deposition or deposition
transcript, and
(d) a list of documents or physical evidence she expects to present.
3. Scope (a.k.a. What You Can Get If You Ask For It) (Rule 26(b))
a. In federal court, the discoverability of information is dictated by the five criteria discussed below:
Relevance, Privilege, Work Product, Undue Burden, and Experts. One way to remember these
criteria is through the sentence: Discovery: For when you Really Want Private Unknown Evidence.
b. Relevance
(1) Parties are permitted discovery into any matter that is relevant to any claim or defense.
Information is relevant if it is likely to make any fact in dispute more or less likely to be true,
regardless of whether the information would be admissible at trial.
EXAMPLE: P sued D for injuries she suffered in a car accident allegedly resulting from Ds
negligence. P sought from D a record of his most recent eye examination as well as any record
suggesting that he abused his wife. The record of eye exam is relevant because it may shed
light on whether D was negligent, but the record of abuse is not relevant because it can have no
bearing on whether D was negligent.
c. Work Product
(1) Even if information is relevant, it might still be undiscoverable because it is work product.
(2) Work product is a document or other tangible object created by a party or partys attorney in

26
anticipation of litigation.
(a) Common examples of work product include a post-incident description of events or records
of communications with witnesses (including expert witnesses).
Exception: Work product is still discoverable in two particular situations:
(a) A party can always obtain a statement that it has made. Thus, if P, soon after a car
accident, briefly speaks with Ds attorney about the accident and the attorney takes notes
on the conversation, P can obtain the notes even though they are otherwise work product.
(b) A party can obtain work product for which it has a substantial need and cant otherwise
obtain without substantial hardship. If a court orders disclosure of work product for this
reason, it must make every effort to keep secret the authors litigation strategy or mental
processes
EXAMPLE: D drives drunk, kills a cyclist, and then leaves the scene of the crime. After
coming home that night, D records the entire event in his diary, as he does before going to
bed every night. His diary entry that evening is not work product because it was not
prepared in anticipation of litigation. Rather it was something he did every night. If,
however, D did not maintain a diary and decided to record the events of the evening
because he was worried about a future lawsuit, that information would be work product and
likely non-discoverable. If D was later sued, P might still be able to obtain the notes if she
can show that the notes contain information essential to her case that would otherwise be
difficult to get, such as Ds description of the moments before the accident (assuming there
were no other witnesses). Even if P can obtain work product in this way, the court will make
sure Ds mental impressions of the case are kept secret. Thus, if D states that my best
defense in a lawsuit will probably be incapacity by reason of drunkenness, but Im not sure
I can prove that I was drunk, the court will forbid disclosure of that information.
d. Privilege
(1) Even if information is relevant to a claim or defense, and not work product, it may still be
protected from discovery if it is privileged. The most common privilege is the attorney-client
privilege (which covers communications between an attorney and client for the purposes of
obtaining or rendering legal advice). Many other privileges exist, however, including the marital
privilege and state secrets privilege.
EXAMPLE: As in the above example, D drives drunk, kills a cyclist, and then leaves the scene
of the crime. After coming home that night, D records the entire event in his diary, as he does
before going to bed every night. P, the cyclists personal representative, later files suit against D,
and D sought the advice of an attorney. The attorney told D to write down everything that
happened that night. D wrote a memo and sent it to the attorney. In response to Ps request for
information pertaining to the events of that evening, D must turn over his diary entry but not the
memo.
EXAMPLE: P sued D, an airline, for injuries she suffered in a plane crash. During discovery, P
sought information from the United States government pertaining to the airlines compliance with
anti-terrorism regulations. If that information is protected by a state secrets privilege, P may not
obtain discovery, no matter how helpful or relevant to her case it may be.
e. Undue Burden
(1) Even if a party seeks information that is relevant, not work product, and not privileged, the
information may still be undiscoverable if the request imposes an undue burden on the party
from whom the information is sought.
(2) An undue burden exists in any of the following three circumstances:
(a) The discovery sought is unreasonably cumulative or can be obtained from a less
burdensome source or in a less burdensome way.
(b) The party seeking the discovery has already had ample opportunity to obtain the
information.
(c) The burden or expense of the proposed discovery outweighs its benefits, considering the
nature of the evidence sought, amount in controversy, and the parties resources.
EXAMPLE: P sued Small Business for fraud, claiming $80,000. P learned that X, an
employee of Small Business, will testify at trial. P wishes to collect evidence to impeach X
when X testifies and thus asks for a large quantity of Xs email communications for the past
7 years. To comply with this request, Small Business will have to spend $65,000 on data
recovery services. Assuming the impeachment evidence is collateral to the central issue in
the case, a court would likely find this request to impose an undue burden. The amount at
stake is only $80,000; and it makes little sense to spend $65,000 in an effort to litigate an
$80,000 claim, especially when the evidence sought is purely for the purpose of
impeachment.
f. Experts
(1) Even if a party seeks information that is relevant, not work product, non-privileged, and does not

27
impose an undue burden, the information may still be undiscoverable if the request seeks
information pertaining to certain types of experts.
(2) Non-Testifying Expert: Parties often hire experts simply to assess the merits of a case and have
no intention of calling the expert to testify at trial. Any information pertaining to the opinion of
these experts is undiscoverable unless the party making the request has an extraordinary need
and has no other way to obtain such information. This situation will arise in the rare
circumstance where there are only a few experts in the world qualified to analyze a particular
matter and the opposing party has already procured the opinion of those experts.
(3) Testifying Expert: Unlike a non-testifying expert, the opinions held by experts who are expected
to testify at trial are discoverablebut only to a limited extent. Besides the information contained
in the expert report (which is a mandatory disclosure), a party can also obtain:
(a) Communications relating to compensation for the experts study or testimony
(b) Communications relating to the data provided by the attorney to the expert;
(c) Communications relating to any assumption the attorney asked the expert to make in
developing the expert opinion.
4. Methods (a.k.a. How to Get Something That Youre Allowed to Get)
a. Even if the information sought is within the scope of discovery defined above, the requesting party
may only obtain the information if she adheres to the specified methods of discovery.
b. Interrogatories (Rule 33)
(1) Definition: An interrogatory is simply a question submitted in writing to any other party. Each
party may submit up to 25 questions on any other party. The interrogatory can seek facts about
the case (Were you wearing your glasses at the time of the accident?) but can also seek
contentions (Do you contend that the plaintiff was contributorily negligent?).
(2) Response:
(a) A party served with interrogatories has 30 days to respond.
(b) The response must be in writing, and if the recipient has any objections to the questions
(e.g., the questions seek irrelevant or privileged information), the objections must be stated
with specificity.
(c) If particular questions can be answered by reviewing documents, and the burden of
reviewing those documents will be roughly equal for the requesting party, the answering
party can simply provide the requesting party access to those documents
c. Requests for Admission (Rule 36)
(1) Definition: Requests for admission are used to resolve certain issues prior to trial. Any party can
submit to any other party a request to admit the truth of any fact (or a facts application to law)
as well as the genuineness of any document. If the matter is admitted, it is deemed conclusively
established for the purposes of the litigation. Parties often use requests for admission in
preparation for trial or motions for summary judgment because they obviate the need to prove
uncontested elements of a case.
(2) Response:
(a) A party served with requests for admission has 30 days to respond.
(b) The responding party may admit the contention, deny it, or state that it has made a
reasonable attempt to ascertain the truth of the matter but lacks sufficient information to
admit or deny it
(c) The responding party can object to questions as outside the scope of discovery, but cannot
simply claim that the matter is properly resolved by a jury.
d. Requests for Production of Documents, Tangible Items, or Access to Evidence
(1) Definition: A request for production of documents (whether maintained in physical or electronic
format), tangible items, or access to evidence is exactly what it sounds likean effort to get
certain forms of evidence. Most requests in this area will be for documents, but if a party would
like to inspect a particular object (e.g., the damaged bumper on the plaintiffs car) or enter real
property (e.g., to assess the location of a handrail), those requests can be made as well.
(2) Response:
(a) A party served with such a request must respond within 30 days and may object to certain
requests as outside the scope of discovery.
(b) When asked for documents, the responding party must provide them as they are
maintained in the usual course of business and label them to correspond to the request.
(c) When asked for electronically stored information (ESI), the responding party may produce
ESI in the form in which it is normally maintained, or may produce copies of documents
stored as ESI.
e. Requests for Mental or Physical Examination (Rule 35)
(1) Definition: A request for a mental or physical examination is used when a partys mental or
physical state is an issue in a case. Thus, if P sues D for causing her lower back pain, D can
have P examined by a licensed physician to determine the existence and extent of such pain.

28
Importantly, this type of discovery is available only upon court order. Most other types of
discovery (e.g., interrogatories, requests for admission, depositions, etc.) may be served on
parties without first seeking permission of the court.
(2) Response: When ordered by a court to submit to such an examination, a party must do so.
However, the examiner must prepare a report detailing the result of her examination and provide
it to any party who requests it.
f. Depositions (Rules 27, 30, 31)
(1) Definition:
(a) A deposition is used to ask questions of a witness, usually in live setting. (Depositions may
be had through writing, but written depositions are rarely used.)
(b) Each party may depose 10 witnesses, but may obtain a court order if further depositions
are necessary.
(2) Timing: Most depositions occur during the pendency of the lawsuit. Like other forms of
discovery, these depositions cannot be conducted until after the 26(f) conference. In rare cases,
a party may wish to depose a witness before a lawsuit is even filed. These are known as
depositions to perpetuate testimony and are available only if all expected opposing parties are
provided an opportunity to be present at the deposition and ask questions
EXAMPLE: P and D are involved in a car accident. P has not yet filed suit because she is
conducting research into the circumstances of the accident. During this research, she learns
that Ethel, a 98-year-old woman, witnessed the accident. P is worried that Ethel may pass away
before P files suit or takes the case to trial. P may depose Ethel and, if Ethel is ultimately
unavailable for trial, use that deposition at trialprovided that P provided D an opportunity to be
at Ethels deposition.
(3) Use in Hearing or at Trial: A party may use a deposition at a hearing or trial as long as the party
against whom the deposition is used had a reasonable opportunity to be present at the
deposition, and only according to the following conditions:
(a) A deposition of a party or partys designee can be used for any purpose.
(b) A deposition of a non-party can be used
1) to impeach the deponent, or,
2) if the deponent is unavailable (e.g., by reason of death, infirmity, or disappearance),
the deposition can be used for any purpose.
g. Subpoenas
(1) Definition: Subpoenas are used to obtain discovery from non-parties.
(a) There are two types of subpoenas:
1) subpoenas duces tecum (a demand for documents) and
2) subpoenas ad testificatum (a demand for testimony).
(b) In this way, subpoenas are a sort of hybrid discovery tool; they are tools in their own right,
but they allow you to use other tools (requests for documents and depositions) to discover
information from non-parties.
(2) Response: A person served with a subpoena may object to the subpoena as outside the scope
of discovery, or that it requires the person to travel more than 100 miles from where she resides
or works.
5. Enforcement (a.k.a. What to Do When You Dont Get What You Asked For)
a. If they sort of comply
(1) Suppose a person presented with a request for discovery (or, for non-parties, a subpoena)
responds but refuses to answer certain questions or provide certain documents, citing privilege
or some other excuse. What is the requesting party to do? The Rules dictate the following
process.
(a) Motion to Compel: The first thing to do is file a motion to compel the person to comply with
the discovery request. Importantly, this motion may only be made after the movant has in
good faith attempted to confer with the person resisting discovery to see if judicial
intervention can be avoided.
1) If the court grants the motion, the movant is entitled to the legal fees and expenses
associated with making the motion.
2) If the court denies the motion, the non-movant may be entitled to fees, but only if the
motion was not substantially justified.
(b) Sanctions: What if, even after a motion to compel has been granted, the person subject to
the order still refuses to permit discovery? At that point, the person is subject to sanctions,
including:
1) a court order declaring that the facts sought are established in favor of the requesting
party,
2) a court order prohibiting the disobedient party from presenting certain claims or
defenses,

29
3) a stay or dismissal of the entire action, or
4) an order of contempt.
b. If they dont comply at all
(1) What happens if a discovery request or subpoena is presented and the recipient simply ignores
it? That is, instead of showing up at a deposition and refusing to answer certain questions based
on attorney-client privilege, the deponent simply never shows up. If this occurs, the requesting
party can immediately seek sanctions and need not start with a motion to compel.
(2) All of the sanctions listed above are available, with the exception of an order of contempt.
(Contempt orders can only be used where a party has violated a court order; thus, without an
order compelling discovery, contempt is inappropriate.)
IV. PRETRIAL ADJUDICATION

A. VOLUNTARY DISMISSALS
1. Sometimes a claimant wants to stop litigating her claim, either because she thinks it has no merit, because
it is too expensive or arduous to litigate, or because the defendant has offered to settle the claim. The rules
below define when voluntary dismissal is available and what effect it may have on future litigation.
2. Dismissal Options
a. Option 1: A claimant can unilaterally dismiss her suit simply by filing a notice of dismissal anytime
before the defendant has filed an answer or motion for summary judgment
b. Option 2: If a claimant cannot unilaterally dismiss (or simply chooses not to), she can dismiss her
claim by filing a stipulation of dismissal signed by all parties. This is what typically happens when a
case settles.
c. Option 3: If a claimant cannot dismiss under options 1 or 2 (or chooses not to), she can dismiss only
with the permission of the court. Generally, a court will grant permission, but if the defendant has filed
a counterclaim, the court will not allow dismissal unless the counterclaim can remain pending for
independent adjudication
3. Prejudice on Future Litigation
a. May the plaintiff, after dismissing her claim, re-litigate it? The plaintiffs ability to re-litigate will depend
on whether the claim was dismissed with prejudice or without prejudice.
b. First, if the notice, stipulation, or court order specifies that the dismissal is with or without prejudice,
the plaintiff is bound by that specification.
c. Second, if the notice or stipulation does not address the issue, the dismissal is presumed to be
without prejudice for the first dismissal. Subsequent dismissals are presumed to be with prejudice.
EXAMPLE: P sues D and has him served with process the same day. Before filing anything with the
court, D calls P and apologizes for the harm he has caused her. P decides to file a notice of dismissal
but does not specify in the dismissal whether the dismissal is with or without prejudice. Ps dismissal
is permissible.
EXAMPLE: After P dismissed her suit, D changed his tune. He retracted his apology and bragged to
his friends how he tricked P into dismissing her suit. P re-filed her suit, but D argued that her new suit
is barred by claim preclusion (a doctrine barring re-litigation of claims that we will discuss later). Ds
argument will fail because Ps voluntary dismissal, owing to its silence on the issue of prejudice, is
presumed to be without prejudice.
EXAMPLE: In this second lawsuit, D filed an answer, and the case proceeded through discovery. P
was disappointed in the facts she was able to discover and did not want to put any more effort into
the suit. She asks D to sign a stipulation of dismissal dismissing the suit without prejudice, but D
refuses to sign it. She therefore asks the court to dismiss the suit. The court obliges, but does not
specify in its order whether the dismissal is with or without prejudice. This dismissal is presumed to
be with prejudice because it is Ps second dismissal.

B. INVOLUNTARY DISMISSALS
1. If the plaintiff fails to prosecute her case or refuses to comply with a court order or any of the Federal Rules
of Civil Procedure, a defendant may move to dismiss the action. A dismissal issued in this circumstance is
always with prejudice. Note that this type of dismissal is one of the sanctions for discovery infractions
discussed above.

C. DEFAULT JUDGMENTS
1. What if P sues D and has him properly served with process, but D never responds? If that happens, D will
be subject to a default judgment. Default judgments involve two steps:
a. the entry of default; and
b. the entry of the default judgment
2. Entry of Default
a. If a defending party fails to defend the action (usually by not responding to the complaint within the
appropriate time), the clerk must enter a default.
30
3. Default Judgment
a. Only after a default has been entered may a default judgment issue. How a default judgment issues
depends on the nature of the plaintiffs claims.
b. Entry by Clerk
(1) If the plaintiffs claim is for a sum certain (or a sum that can be made certain by computation),
and the plaintiff requests the default judgment, the clerk must enter a default judgment and
assess damages and costs against the defendant. Clerks may not enter default judgments
against minors or incompetent persons
EXAMPLE: P loaned D $100,000 at an interest rate at 5% per year, with repayment due one
year from the date the loan was made. D never repaid the money, so P sued D and had him
properly served. D never responded. The clerk may enter a default as well as a default
judgment. Ps claim is for a sum certain (or a sum that can be made certain by computation) and
is not lodged against a minor or incompetent person.
EXAMPLE: P was injured in a car accident involving D. P sued D for $100,000 for injuries she
sustained to her back, including $35,000 in medical costs and $65,000 in pain and suffering. P
had D properly served but D never responded. The clerk can enter a default but not a default
judgment. Although D is not a minor or incompetent person, P has not sued D for a sum certain.
True, $100,000 is a certain amount, but it is not certainly the amount of harm P suffered.
c. Entry by the Court
(1) If the clerk cannot enter a default judgment, it must be entered by the court. The court will not
issue the judgment automatically; the plaintiff must make an application to the court.
(2) Minor or Incompetent Person: If the defendant is a minor or incompetent person, the court will
issue a judgment only if the defendant is represented by a guardian or some other competent
person
(3) Appearance but Later Default: If the defendant initially appeared in the case but later refused to
defend the action, the court may issue a default judgment only if the defendant has been served
with written notice of the application for default seven days before a hearing on the application.
(4) Hearings: The judge may hold hearings to ascertain the appropriate amount of damages or any
other matter the judge deems appropriate.
4. Relief From Entry of Default and Default Judgment
a. If a clerk enters a default, the court may set it aside for good cause. If the clerk or court enters a
default judgment, the court may set it aside in accordance with Rule 60(b)a post-trial motion
discussed later in this outline.
EXAMPLE: P sued D, and D drafted an answer denying Ps allegations. On the way to the
courthouse to file the answer, D had a heart attack. Six days later, D was able to have his answer
properly filed, but the clerk had already entered a default. D may ask the court to set aside the
default, and the court will likely do so.

D. MOTION TO DISMISS
1. Defined
a. A motion to dismiss is a motion filed by a defending party seeking the dismissal of a claim filed
against him. Motions to dismiss are usually filed by defendants, but a plaintiff can file a motion to
dismiss if she is sued on a counterclaim. The same goes for third party defendants (sued on
impleader claims) or co-defendants (sued on cross claims).
2. Timing
a. A motion to dismiss must be filed before an answer is filed. Given that the time within which to file an
answer is either 21 days (where process was served) or 60 days (where service of process was
waived), a motion to dismiss must be filed within 21 or 60 days.
3. Grounds
a. A motion to dismiss may be made on any of the following grounds.
b. Lack of subject matter jurisdiction
EXAMPLE: P sued D, who is non-diverse, for injuries she sustained in a car accident. Assuming
there is no federal law addressing car accidents, D should file a motion to dismiss arguing that the
court lacks subject matter jurisdiction over the dispute.
c. Lack of personal jurisdiction

EXAMPLE: P traveled to New York for vacation, where she was injured by a New York resident. She
sued the resident in her home state of Oklahoma. The New York resident should file a motion to
dismiss challenging the courts personal jurisdiction over him.
d. Improper venue

EXAMPLE: P traveled to New York for vacation, where she was injured by a New York resident. She
brought suit in the Southern District of New York because that was the most convenient venue for her.

31
D does not reside in the Southern District, and the cause of action did not arise there. D should move
to dismiss the suit for improper venue.
e. Insufficient process

EXAMPLE: P sued D and chose to have him served by flushing the process down the toileta
method approved by the courts of the state in which P sued. D should move to dismiss based on
insufficient process because that method of service was not reasonably calculated to apprise him of
the claims against him.
f. Insufficient service of process

EXAMPLE: P sued D, and the process server left process at Ds home with a friend staying at the
house for the weekend. Although D received the process, that method of service was impermissible
under state and federal rules. D should move to dismiss based on insufficient service of process.
g. Failure to state a claim upon which relief can be granted
EXAMPLE: P sued D for speaking French in a public place. D should file a motion to dismiss for
failure to state a claim because, regardless of whether D was speaking French, it is not unlawful to
speak French in a public place.
A couple further notes are in order for this type of motion to dismiss
(1) First, a court assessing a motion to dismiss always assumes the claimants allegations are true
i.e., that D really was speaking French.
(2) Second, dont forget (as we noted in the section on drafting the complaint) that the claimants
allegations must be plausible to survive this type of motion to dismiss. A claim need not be
probable to be plausible, but it does need to have enough facts supporting it for the judge to
find that the plaintiff is onto something, or rather, is not merely guessing that the defendant
committed a wrong.
EXAMPLE: P buys gas every week a couple blocks from her house. There are two gas stations
there, one on each corner. P thinks it is odd that the prices at the stations are nearly always
identical. She believes they are colluding and sues them both for a violation of federal trade law.
To survive a motion to dismiss for failure to state a claim, Ps complaint must contain a factual
predicate far more substantial than the simple claim that the prices are usually the same.
h. Failure to join an indispensable party
EXAMPLE: P sued D for failing to return a painting she had leased him. The painting is now in the
hands of X, a person to whom D sub-leased the painting. X is a necessary party. D should move to
dismiss the suit based on Ps failure to join X in the suit.
4. Waiver
a. The federal rules require certain defenses to be raised at certain times. If they are not raised at that
time, the party possessing the defense waives the right to raise it. These defenses might thus be
called use em or lose em defenses, meaning that if the defending party makes a motion to dismiss,
but does not include the defense, the party forfeits the defense. Note that if a party does not file a
motion to dismiss but instead files an answer, the defense must be raised in that pleading or else it is
waived.
b. Use em or Lose em: In the First Response
(1) If a party wishes to defend the action against him on any of the grounds below, and he files a
motion to dismiss, he must include the defenses in the motion or else forfeit them. (Note that if a
party files an answer instead of a motion to dismiss, these defenses must also be raised in that
pleading or else are forfeited.)
(a) Lack of personal jurisdiction
(b) Improver venue
(c) Insufficient process
(d) Insufficient service process
c. Use em or Lose em: Before the Trial Ends
(1) If a party wishes to defend the action against him on any of the grounds below, he must raise
them anytime before the trial ends, or else forfeit them.
(a) Failure to state a claim upon which relief can be granted.
(b) Failure to join a necessary party
d. Never Lost
(1) If a party believes that the court lacks subject matter jurisdiction over a claim, it may raise this
defense at any time, even on appeal. This defense is never forfeited
EXAMPLE: P sues D for injuries she sustained in a car accident. D files a motion to dismiss
challenging venue, which is denied. D then files a motion to dismiss challenging personal
jurisdiction. This motion will be denied because D forfeited the defense by not raising it in his
initial motion.
EXAMPLE: After Ds motion to dismiss for lack of PJ is denied, he files an answer listing the

32
defense of insufficient service of process. D may not raise that defense because he failed to
raise it in the initial motion to dismiss.
EXAMPLE: During the litigation, D argues that Ps complaint fails to state a claim upon which
relief can be granted. The court may entertain that argument because D did not waive it, even
though he did not include it in his initial motion to dismiss.
EXAMPLE: D loses at trial and decides to appeal. On appeal, D argues that the court lacked
subject matter jurisdiction over the suitan argument D has never made before. D did not waive
that argument, and the court of appeals must consider it.

E. MOTIONS FOR SUMMARY JUDGMENT


1. Defined
a. A motion for summary judgment is a motion challenging a claim or defense on the merits. In making
the motion, the movant is essentially arguing, If we went to trial right now, the jury would undoubtedly
rule in my favor on this claim or defense.
b. Using the language of the Federal Rules, the movant is arguing that there is no genuine dispute as
to any material fact and I am entitled to judgment as a matter of law.
2. Analysis
a. Step 1
(1) When a party moves for summary judgment, the first step is for the court to assess the
arguments advanced by the movant.
(a) The court considers whether the movant has shownbased on the movants arguments
alonethat the non-movant lacks sufficient facts and/or law to prevail on the claim or
defense in question.
(b) The movant can do this in either of two ways:
1) by pointing outusing citations to the recordholes in the opposing partys claims or
defenses, or
2) by adducing new evidence demonstrating that the claim or defense cannot be true.
NOTE: In considering these arguments, the court will not question the credibility of D
or Ds witnesses.
EXAMPLE: P sues D sue for injuries she sustained in a car accident. During
discovery, D asks for information pertaining to Ps injuries, and she supplies none. D
should move for summary judgment and cite to the record showing Ps lack of
evidence. In the words of the rule, D is arguing, There is no dispute that P was not
injured and the law says that non-injured persons have no right to relief. Thus, I win.
EXAMPLE: P sues D for injuries she sustained in a car accident. During discovery, D
asks for information pertaining to Ps injuries, and she supplies a report from a
voodoo doctor claiming that a nasty spirit has entered her body as a result of the
accident. D should move for summary judgment and cite to this report in the record
as Ps only evidence of injury. D is arguing, Ps evidence fails to create any genuine
dispute of material fact. The evidence is so weak that no reasonable jury could find
that P was injured, and the law says that non-injured persons have no right to relief.
Thus, I win.
EXAMPLE: P sues D for injuries she sustained in a car accident. D had three friends
with him in that car at the time, each of whom believe that the light was green in his
favor. D should move for summary judgment and proffer affidavits from these
witnesses. D is arguing, I have evidence showing that I was not negligent, and the
law says non-negligent people are not liable. Thus, I win.
b. Step 2
(1) If the movant has cleared the Step 1 hurdle, the court then turns to the non-movants response.
(If the movant has not cleared the hurdle, the court simply denies the motion and doesnt even
need to turn to the non-movants response.) To survive summary judgment, the non-movant
must adduce evidence sufficient for a reasonable jury to find in her favor on a claim or defense
(a) In determining whether the non-movant has done this, the court will look only at the non-
movants evidence (i.e., the court does not weigh the evidence against the movants) and
will assume all of her witnesses are truth tellers (i.e., the court does not consider credibility
issues).
(b) If the non-movant does not produce evidence but merely refers the court to her pleadings
(a.k.a. sitting on her pleadings), the court will grant the motion. (If the pleadings are filed
under oath, however, they amount to an affidavit, which does have evidentiary value.)
EXAMPLE: P sues D for injuries she sustained in a car accident. D moved for summary
judgment after discovering that P has never sought the care of a licensed physicianthus
indicating that P never suffered an injury. In response, P supplies an affidavit from her

33
uncle, a physician licensed in England, which describes Ps injuries and his treatment plan
for her. Upon this affidavit, the court should probably deny the motion for summary
judgment because such evidence is sufficient for a reasonable jury to find that P suffered
an injury. In other words, there is a genuine dispute of material fact for the jury to resolve.
EXAMPLE: P sues D for injuries she sustained in a car accident. D moves for summary
judgment and as evidence in his favor, offers the affidavit of 100 nuns who were picnicking
near the scene at the time of the accident. The nuns unanimously, and with characteristic
grace, state that the light was green in Ds favor at the time of the accident. P responds
with a single affidavit from Damian, a devil in human form and a known liar, stating that the
light was green in Ps favor at the time of the accident. The court should deny the motion
because Ps evidence, taken alone (i.e., not weighed against Ds evidence) and assumed
to be true, is sufficient for a jury to find in her favor. P has, in other words, shown a genuine
dispute of material fact.
c. Who can make the motion?
(1) Plaintiffs or defendants can move for summary judgment. However, defendants make summary
judgment motions far more often than plaintiffs. This is because the plaintiff, to prevail on her
own motion for summary judgment, must adduce evidence sufficient for a reasonable jury to find
in her favor on every element of her claim and hope the defendant is able to respond with no
evidence on any element of the claim.
NOTE: One summary judgment argument commonly used by defendantspointing out, with
citations to the record, the gaps in the claimants evidenceis unavailable to plaintiffs when they
move for summary judgment.
d. Materials used
(1) Summary judgment concerns the sufficiency of the evidence in the case. How is this evidence
presented to the court in a motion? The rule allows the parties to present, and the court to refer
to on its own, the pleadings, discovery documents, materials produced in mandatory disclosure,
and affidavits.
(2) Importantly, the information offered to the court needs to be admissible at trial, even if it is not
presently in a form that would be admissible
EXAMPLE: P sues D, and D moves for summary judgment. P responds with an affidavit from a
witness stating that she saw D drive through the red light and strike P and that a friend told
me last week that he saw D run through that red light, too. The court will consider the witnesss
first statement but not the second. The first statement contains admissible evidence even
though affidavits, on their own, are not admissible at trial. The second statement is hearsay and
not admissible, even if presented via live testimony.
e. When may the motion be made?
(1) Unless a local rule or court order says otherwise, a party may move for summary judgment
anytime before 30 days after the close of discovery.
V. TRIALS AND POST TRIAL MOTIONS

A. TRIAL BY JURY
1. The Scope of the Right
a. The Seventh Amendment of the United States Constitution grants litigants in federal court the right to
a trial by jury [i]n suits at common law, where the value in controversy shall exceed twenty dollars.
The $20 requirement is pretty easy to figure out, but what does it mean for a suit to be at common
law? There are two considerations: the nature of the remedy, and the nature of the claim.
b. The Nature of the Remedy
(1) In 1791, so-called common law courts heard both common law and statutory actions, but only
had the power to issue damages, not equitable relief (such as injunctions, specific performance,
rescission, or reformation). Thus, the Seventh Amendment guarantees a right to a jury where
monetary relief, rather than injunctive relief, is sought. If both forms of relief are sought, the right
to a jury exists for any issue of fact underlying a damages claimeven if the resolution of that
issue may also support injunctive relief
EXAMPLE: P sued D for creating a nuisance by disposing of chemicals in the river. P sought
damages for damage to her garden. Either party is entitled to demand a jury trial.
EXAMPLE: P sued D for creating a nuisance by disposing of chemicals in the river. P sought an
injunction barring further disposal. Neither party has a right to demand a jury (though Congress
is free to provide one if it so desires).
EXAMPLE: P sued D for creating a nuisance by disposing of chemicals in the river. She sought
monetary relief for her lost vegetables and injunctive relief barring further disposal. One issue in
the case is whether the chemical is harmful to Ps health. Either party may demand a jury on this
issue because it is necessary to resolving Ps demand for damages. The resolution of this issue
will also support an order for injunctive relief, but that does not defeat the right to a jury trial.
34
c. The Nature of the Claim
(1) Just because a plaintiff seeks damages does not guarantee that a right to trial by jury will exist.
The Supreme Court has interpreted the Seventh Amendment to guarantee a jury trial only for
the types of claims that would have been actionable in 1791 when the Amendment was ratified.
Because times have changed, there are many suits filed today that would not have been
cognizable in a common law court of 1791, regardless of the remedy sought.
(2) For example, common law judges in 1791 would have laughed themselves silly if a union tried
to file suit against an employer for a breach of good faith and fair dealing. Whats this thing
called a union, and what does fairness have to do with employment, they would have asked.
Does that mean there is no right to a jury in such cases? The answer depends on whether the
cause of action can be fairly analogized to a claim that would have been actionable in a 1791
common law court.
(a) In a case involving a suit of the sort just described, the Supreme Court held that that suit
was most similar to an equitable action for breach of fiduciary duty (which would not entitle
the parties to a jury) but that the plaintiffs claim for damages tipped the scales in favor of
making a jury available.
(b) The Supreme Courts analogy jurisprudence in this area is so discretionary that there is
little more guidance to be provided here. Do remember, however, the key concept here: if a
claim, though dressed up in modern garb, is functionally similar to a claim that would have
been actionable in a 1791 common law court, any party to that claim may demand a jury.
2. Exercising the Right to a Jury
a. Who can exercise the right?
(1) Any party can exercise the right to a jury trial. Thus, if a suit involves ten parties and only one
party wants a jury trial, that party can force the other nine parties to try the entire suit to a jury.
b. Demand
(1) To obtain a trial by jury, the party desiring it must file with the court and serve on the other
parties a written demand for a jury trial.
(2) The demand must be filed no later than 14 days after the last pleading directed to the [jury-
eligible] issue is filed.
(a) Remember that pleadings is a term of art in the Federal Rules; they include mainly
complaints and answers, but also include a reply to an answer if the court permits it.
c. Waiver
(1) If a written demand is not timely filed and served, the right to a trial by jury is forfeited.
3. Jury Composition
a. Size
(1) Juries can be composed of six to twelve members.
b. Selection
(1) The process through which jurors are selected is called voir dire. Potential jurors are questioned
and evaluated by the judge and parties to determine their fitness to serve. If these evaluations
suggest that the juror is unfit to serve, or simply not preferable to either party, the juror can be
struck in the following ways.
(2) Challenges for Cause: During voir dire, any party may ask the court to dismiss a potential juror
because the juror is unfit to serve. The most common ground for unfitness is the jurors
confessed or likely bias towards a party or issue in the case. So if a juror responds to a question
with, well, yes, as it turns out, I was also discriminated against by that dastardly, good-for-
nothing, going-straight-to-hell defendant over there, the juror is subject to dismissal for cause.
Each party has an unlimited number of challenges for cause, but must articulate the nature of
the jurors unfitness in each instance.
(3) Peremptory Challenge: In addition to challenges for cause, jurors can also be removed through
a peremptory challenge.
(a) Each party has three such challenges and when exercised, the juror is automatically
dismissed.
(b) In making a peremptory challenge, a party need not articulate its reason for dismissal
unless the strikes give rise to an inference of racial or gender discrimination.
1) If that is the case, the striking party must provide a nondiscriminatory explanation for
the strikes or else rescind its strike of each juror involved
4. Jury Instructions
a. Before deliberating, the judge instructs the jury on the law applicable to the claims alleged.
b. Any party may propose particular instructions no later than the close of all evidence.
c. Any party may also object to any instruction, but most do so before the jury begins its deliberations.
5. Jury Verdicts
a. Unanimity
(1) Unless the parties stipulate otherwise, jury verdicts must be unanimous.

35
b. Types of Verdicts
(1) General Verdict: The court may direct the jury to return a general verdict, which is simply a
statement that P wins or D wins.
(2) Special Verdict: In contrast, the court may direct the jury to answer specific questions of fact. For
example, a court may ask the jury to decide whether P was contributorily negligent in failing to
wear her seatbelt and whether D was impaired in his judgment because of his use of a
sleeping aid.
(3) General Verdict With Specific Questions: As a third option, the court may order the jury to return
a general verdict (is D liable to P?) but also to answer specific questions (was D impaired
while driving?). If the verdict and answers to the questions are inconsistent, the judge can:
(a) Order a new trial
(b) Order the jury to re-deliberate, or
(c) Enter the verdict appropriate to the specific answers.

B. BENCH TRIAL
1. Findings and Conclusions
a. If a claim is tried before a judge rather than a jury, the court must find the facts specially (meaning
that the court must state its finding with regard to facts in dispute) and state its conclusions of law
separately. In a sense, the judge acting as fact-finder is required to issue special verdicts.
NOTE: This requirement only applies to trials; judges need not make such findings or statements of
law on motions to dismiss or for summary judgment.
2. Partial Judgments
a. During trial, if a party has been fully heard on an issue, and the court determines that the party lacks
sufficient evidence to prevail on a particular claim, the court can enter judgment on that claim without
entering judgment on other claims. Entering partial judgment in this way facilitates interlocutory
appealsa topic we will cover soon.

C. MOTION FOR JUDGMENT AS A MATTER OF LAW (RULE 50(A))


1. During a jury trial, the court may enter (on its own accord or upon the motion of a party) a judgment as a
matter of law (JML) on a particular claim.
2. A JML is appropriate if:
a. the party against whom the judgment is entered has been fully heard on an issue, and
b. the party lacks sufficient evidence to prevail on an issue necessary to a particular claim or defense.
This standard is, in essence, the same as the summary judgment standard. A court will thus assume
the non-movants witnesses are all truth tellers and not weigh the evidence.
EXAMPLE: P sues D for injuries she sustained as a passenger in his car. To prevail at trial, P needs
to show that D was negligent and that his negligence caused her injury. During trial, P put on
evidence that D was negligent, but failed to put on any evidence that she suffered injury. After P has
been fully heard on her claim, D moves for JML. The court may grant the motion because P lacks
evidence sufficient for a jury to find that she was injuredan issue necessary to prevail on her claim.

D. POST-TRIAL MOTIONS
1. Suppose your case goes to trial and you lose. What can you do (besides appeal)? You have several
options, each of which is discussed below.
2. Renewed Motion for Judgment as a Matter of Law (Rule 50(b))
a. Defined:
(1) If a party files an unsuccessful motion for a JML during trial, and then loses at trial, the party
may file a renewed motion for judgment as a matter of law. This motion makes the same
argument as an ordinary JML and is decided according to the same standard. Note that the title
to this motion is telling: a renewed motion for judgment as a matter of law presupposes that a
JML motion was made at trial. If no such motion was made a party may not make a renewed
motion for JML.
b. Joint motion for a new trial
(1) When making a renewed motion for JML, the movant may move jointly or in the alternative for a
new trial (a motion we will discuss shortly).
c. Timing:
(1) A renewed motion for JML must be filed within 28 days of judgment.
(2) Note that the date of judgment is not necessarily the date of the verdict. The date of the verdict
is the date when the jury delivers its decision on the contested factual issues. The date of
judgment is the date when the court actually declares one party or the other the winner.
d. Judicial options:
(1) In ruling on a renewed motion for JML, the court may
(a) allow the verdict to stand,

36
(b) enter the opposite verdict, or
(c) order a new trial (even if a motion for new trial was not made).
(2) Note, however, that if the court grants a renewed JML motion, it must also issue a conditional
ruling on any motion for a new trial. This helps with any potential appeals because, if the courts
JML ruling is set aside on appeal, the appeals court can also review the appropriateness of a
new trial.
3. Motion for a New Trial (Rule 59)
a. Defined
(1) A motion for a new trial is exactly what it sounds like: a motion seeking a repeat of the trial. The
old trial is declared a nullity, and the whole process starts over.
b. Grounds
(1) A motion for a new trial may be granted for any of the following reasons:
(a) Against the Great Weight of the Evidence: If the jury returns a verdict that ignores the great
weight of the evidence, a court may order a new trial. Interestingly, the court may take
account of the comparative volume of and credibility of the evidencesomething the court
may not do in a summary judgment motion or a judgment as a matter of law.
(b) Excessive Damages: If a jury returns a verdict with damages that are grossly excessive
or shock the conscience, the court may, upon a motion, order a new trial. The court may
also choose an amount of damages it considers appropriate and give the defendant the
option of paying that amount or submitting to a new trial. Note that there is no remedy in
federal court if the damages are grossly inadequate.
EXAMPLE: P sued D for a broken leg P suffered at Ds amusement park. P sought
$100,000 in compensatory damages, but the jury, for some unknown reason, returned a
verdict in Ps favor for $850,000. A court may order a new trial, or in the alternative, choose
an amount of damages it considers appropriate to the harm suffered, and offer D the
option of paying that to avoid the new trial.
(c) Procedural Error or Misconduct: If the judge, parties, or witnesses violate the rules of trial,
a new trial may be ordered if, and only if:
1) the error or misconduct likely affected the result of the trial; and
2) the party objected to the error when she had the opportunity to do so
3) Examples of errors or misconduct include:
a) Wrongful exclusion of evidence at trial
b) Incorrect jury instructions.
c) Communication between witnesses
d) Ex parte communication with the judge
e) Juror misconduct
(d) Newly Discovered Evidence: If a party discovers evidence after the verdict that could not,
with reasonable diligence, have been discovered before the verdict, a new trial is
warranted as long as the evidence is not solely for the purpose of impeachment and would
likely change the result.
EXAMPLE: P sued D for a car accident and lost at trial. The verdict was reported in the
paper and X, who saw the accident but did not know of the lawsuit, contacted the plaintiff
to provide her with a video of the incident he accidentally captured on his smartphone. P is
entitled to a new trial as long as she had no way to know of Xs video and the video is so
powerful that it would change the result of the trial.
EXAMPLE: P sued D for a car accident and lost at trial. After trial, P discovered that Ds
lone witness was previously convicted of perjury. Even if P had a good excuse for not
finding this evidence sooner, and even if the evidence would have changed the result of
the trial, a new trial is not appropriate because the evidence is presented solely for the
purpose of impeachment.
c. Timing
(1) A motion for a new trial must be filed within 28 days of the entry of judgment.
4. Motion for Relief From Judgment (Rule 60(b))
a. Defined
(1) A motion for relief from judgment asks the court to undo the judgment entered. Such a motion
need not specify the appropriate steps the court should take after the judgment has been
undone; it need only ask for the court to set aside the judgment.
b. Grounds
(1) A motion for relief from judgment may be granted for any of six reasons:
(a) Mistake, Inadvertence, or Excusable Neglect: This ground is often used by litigants against
whom a default judgment has been entered, but it need not be confined to that context.
EXAMPLE: P sued D. D failed to respond and P obtained a default judgment against D. D
learned of the judgment and asked the court for relief from judgment because he has been

37
in a coma for the past two months. The court will very likely grant the motion and allow D to
defend himself in the action.
(b) Newly Discovered Evidence: The discovery of new evidence can be used to support either
a motion for a new trial or relief from judgment. The same analysis applies (the evidence
must not have been discoverable before trial, the evidence must likely change the result,
and the evidence must not be for the purposes of impeachment). Whether to move for a
new trial or relief from judgment will depend on the timing of the motion (see below for
more information on timing).
(c) Fraud, Misrepresentation, or Misconduct by a Party: If a party withholds key evidence
during discovery, bribes a witness, or makes false statements to the court or other parties,
a party subject to an adverse judgment may seek relief from that judgment.
(d) Judgment Is Void: If the judgment entered is void because the court lacked subject matter
jurisdiction or personal jurisdiction, a party subject to the judgment may seek relief from the
judgment.
(e) Judgment Has Been Satisfied: If a party has satisfied the judgment entered against it, the
party may seek to have the judgment discharged so that it is no longer under the
supervision of the court. This type of relief is most often sought in suits involving injunctive
relief.
(f) Any Other Reason That Justifies Relief: This is a catch-all category. There are few
examples of its use, but there is wide agreement that it cant be used when one of the
above five options would apply.
c. Timing
(1) A motion for relief from judgment must be made, first and foremost, within a reasonable time. In
practice, that means that the movant should make the motion promptly after learning of the
grounds for the motion.
(a) However, any motion based on the first three grounds (mistake, new evidence, and fraud)
must be made no later than one year. Thus, even if a party moves for relief from judgment
as soon as she locates crucial new evidence, for example, the motion may not be granted
if it was made more than a year from the final judgment.

VI. APPEALS AND PRECLUSION

A. APPEALS
1. Appellate Courts
a. In almost all instances, a party wishing to appeal a district court order must appeal to the Court of
Appeals for the circuit in which the district court sits.
b. If a party desires to appeal a Court of Appeals order or judgment, the only avenue for appeal is to the
United States Supreme Court. The Supreme Court is not obliged to hear any appeals; it hears
appeals at its discretion.
2. Appellate Jurisdiction
a. Federal circuit court jurisdiction is limited.
b. Federal circuit courts have jurisdiction to hear only appeals seeking review of: (1) a final judgment, (2)
an order pertaining to preliminary injunctive relief, (3) an order pertaining to certification of a class, (4)
an order that has been certified by the district court, or (5) a collateral order.
(1) The orders described in (2), (3), (4), and (5) are sometimes referred to as interlocutory orders
because they are issued during the pendency of the litigation.
c. Final Judgment
(1) A party may appeal a district court order when a final judgment has been issued. A final
judgment is a judicial act that disposes of the entire case. When a final judgment has been
entered, there is nothing left for the court or parties to address. If matters do remain, then
whatever judgment that was entered was apparently not final.
NOTE: The court has discretion to enter a partial final judgmenta final judgment on one
claim but not others. If the court does this, a party may appeal the claim on which that judgment
was entered.
EXAMPLE: P sues D for breach of contract and fraud. D moves for summary judgment on the
fraud claim, but not the contract claim. The court grants the motion. P cannot appeal that ruling
until the court has ruled on the contract claim, unless the court issues a partial judgment on the
fraud claim.
EXAMPLE: P sues D for injuries sustained in an auto accident. D moves to dismiss for lack of
personal jurisdiction. Even though the courts decision may be erroneous, P may not appeal this
decision until the entire litigation (including, perhaps, a full trial) is concluded.
d. Preliminary Injunctive Relief
(1) Any party may immediately appeal a district court order granting or denying a preliminary
38
injunction or temporary restraining order
EXAMPLE: P filed suit against D, a corporation intent on taking over another corporation. P
sought a preliminary injunction barring this take-over. If the court grants the PI, D may
immediately appeal; if the court denies the PI, P may immediately appeal.
e. Class Certification
(1) Any party may immediately appeal a district court decision granting or denying a motion to
certify a class in a putative class action suit.
EXAMPLE: P is the lead plaintiff in a putative class action against a major pharmaceutical
manufacturer. P sought to have a class certified consisting of all persons injured by the use of
the defendants product. The court denied certification. P may appeal this decision without
waiting for a final judgment.
f. Certified Order
(1) A party may immediately appeal an order that the district court has certified for appeal.
Certified orders, confusingly enough, are not necessarily orders pertaining to class certification.
They are orders that a court has issued that it decides should be certified for appeal. Courts
are free to certify an order if three conditions are met:
(a) The order involves a controlling question of law;
(b) The issue of law is one on which there is substantial difference of option; and
(c) An immediate appeal will materially advance the ultimate resolution of the action.
EXAMPLE: P sues D in a product liability action. P seeks documents from D pertaining to Ds
selection of safety featuresdocuments that everyone knows will make or break Ps case. D
refuses to disclose these documents, calling them privileged. P moves to compel and the court
denies the motion in a long, thoughtful opinion that notes a stark circuit split on the issue. If the
court desires, it may certify this order for immediate appeal.
g. Collateral Order
(1) The collateral order doctrine, which is extremely narrow, permits a party to appeal an order if
three conditions are met:
(a) The order pertains to a matter unrelated to the merits (i.e., collateral to the merits);
(b) The order conclusively decides a particular issue; and
(c) Delaying appeal until a final judgment has issued would effectively deny appellate review
of the issue.
EXAMPLE: P sued D, a corporation. State law required plaintiffs suing corporations to post a
bond that would cover the corporations legal expenses in case the lawsuit was later deemed
frivolous. D demanded P post the bond but the court, in an opinion citing Erie, disagreed. The
corporation may immediately appeal the order because the bond issue does not pertain to the
merits, the courts decision on the issue is final, and forcing D to wait until a final judgment is
issued will effectively deprive it of the security the bond is intended to provide.
3. Appellate Review
a. Once an appellate court determines that it has jurisdiction to hear an appeal, it must then determine
how to analyze the lower courts allegedly erroneous decision.
b. Standard of Review
(1) Questions of Law: When an appellant claims that a lower court made an error in its assessment
of the law, the appellate court reviews the lower courts decision de novo
(a) De novo review means that the appellate court grants no deference to the lower court and
addresses the legal issue as if it has never been addressed in the case.
(2) Questions of Fact: When an appellant claims that the lower courtwhether acting through the
judge or the jurymade a mistake on factual matter, the appellate court will affirm the lower
courts finding unless it is clearly erroneous.
(a) It is irrelevant whether the appellate judges might have decided the factual issue if they
were trial judges or members of the jury; their only inquiry is whether the fact finder below
clearly made a mistake of fact. This is a high standard to meet and appellate courts rarely
overturn lower court findings of fact.
(3) Inherently Discretionary Questions: All questions are either questions of law or fact.
Nonetheless, regardless of whether a particular question is one of law or fact, some questions
questions in which trial courts are imbued with significant discretionare reviewed using a
deferential abuse of discretion standard.
(a) Examples of such questions include: whether to permit amendment, whether to exclude
evidence as prejudicial, or whether to bar discovery as an undue burden.
(b) In practice, the abuse of discretion standard is similar to the standard applied to questions
of fact: only if no reasonable judge could have reached the same decision will the decision
be disturbed.
c. Harmless Error
(1) Appellate courts may affirm an erroneous lower court decision if the error did not affect the

39
result. Such errors are deemed harmless.
EXAMPLE: P sued D for injuries she sustained in a car accident. P sought a deposition of Ds
grandmother, who was in Ds car at the time of the accident. The grandmother, who was in poor
health, did not wish to be deposed. The court refused the deposition, calling it an undue burden.
P loses at trial. To prevail on appeal, P must demonstrate not only that the court was wrong in
refusing the deposition, but also that the grandmothers testimony would have changed the
result of the trial.
d. Waiver
(1) An appellant cannot seek relief on appeal if she passed up an opportunity to challenge the
decision at the time the lower court made it. By failing to challenge it at the time, the appellant
waived her right to appeal that issue.
EXAMPLE: P sued D and the case went to trial. The court asked for objections to the jury
instructions but no party voiced any objection. P lost at trial. In reviewing the jury instructions
after the trial, P discovered that the court accidentally inserted the word not in the sentence if
you find that the defendant drove while intoxicated, you must find the defendant not liable. P
may not appeal this error because she had the option to challenge it before and did not exercise
that option.
4. Appellate Procedure
a. In General
(1) In general, an appellant must file a notice of appeal in the district court within 30 days of the
judgment (or within 30 days of the order that is the subject of the appeal). Where the appeal is
based on an order granting or denying class certification the appellant has 14 days to file a
notice of appeal.
b. When Post Trial Motions Have Been Filed
(1) If a post trial motion has been filed (e.g., a renewed motion for judgment as a matter of law, a
motion for new trial, or a motion for relief from judgment), and the court denies the motion, a
new 30 day period begins to run from the date of the denial. If the motion is granted, the
judgment is no longer final and no appeal is permissible unless the order can be certified or
characterized as a collateral order.

B. PRECLUSION
1. Claim Preclusion
a. Defined
(1) The doctrine of claim preclusion (sometimes called res judicata) bars claimants from re-
litigating a case that they have already lost. By definition, the doctrine will arise where there are
two lawsuitsone that has been resolved, and one that is pending.
b. Elements
(1) A plaintiff will be precluded from litigating a claim if the following three elements are satisfied:
(a) Same Parties on the Same Side of the v: The current claim is between the same parties
as a prior case, and each party is positioned on the side of the v as he or she was in the
first suit
(b) Same transaction or occurrence: The current claim arises from the same transaction or
occurrence underlying the prior suit.
(c) Valid Final Judgment on the Merits: The prior suit, to have preclusive effect, must have
concluded in a valid final judgment on the merits. A final judgment, as noted in the
discussion of appeals, is a court order that resolves the entire case in favor of either the
plaintiff or defendant. A final judgment is valid as long as the court had personal
jurisdiction over the defendant. And a final judgment is on the merits if it involved an
inquiry into the merits of the plaintiffs claim.
1) The easiest way to remember whether a judgment is on the merits or not is to
remember the three key instances when a judgment is not on the merits: dismissal for
lack of PJ, SMJ, or venue. All other judgments will be on the merits (e.g., trial verdict,
summary judgment, motion to dismiss for failure to state a claim, and default
judgment). Finally, dont forget that voluntary dismissals can sometimes be with
prejudice or without. A dismissal with prejudice is considered a dismissal on the
merits.
c. Relationship to compulsory counterclaim rule
(1) Recall from our discussion of joinder that defendants not only have the opportunity to bring
counterclaims, but sometimes must bring counterclaims or else forfeit them in future litigation.
Such counterclaims are known as compulsory counterclaims. The compulsory counterclaim
rule is not simply claim preclusion in different dress. Remember, claim preclusion bars claims by
the same P and D, positioned on the same side of the v. If D fails to bring a compulsory
counterclaim in the first suit, and then files it as a plaintiff in a later suit, D has jumped to the

40
other side of the v and claim preclusion will not apply.
EXAMPLE: P was a passenger in a bus owned by D when the bus became involved in an
accident. P sued D for injuries she sustained to her leg but lost in a jury trial. P then sues D
again, this time for the loss of some expensive artwork P had with her on the bus. Ps claim is
barred by claim preclusion because the claim involves the same parties as in the first suit (both
positioned on the identical side of the v), the claim arises from the transaction or occurrence
underlying the first suit (the bus accident), and the first suit ended in a valid final judgment on
the merits (a jury trial).
EXAMPLE: P sued D for injuries she sustained in a bus accident but the suit was dismissed for
lack of personal jurisdiction. P then re-filed the suit in a state where D was subject to personal
jurisdiction. Ps second suit is not precluded because the first suit did not end in a judgment on
the merits.
EXAMPLE: P is injured in a bus accident. She sued the bus company on the theory that the bus
driver was negligent and the bus company is vicariously liable for the drivers negligence. P lost
her suit on summary judgment and then filed a negligence claim against the bus driver. Ps
negligence claim against the driver is not precluded because it is not between the same parties.
EXAMPLE: P sued D for fraud in connection with a contractual relationship. The case went to trial
and the jury held that D did not commit the fraud. Emboldened by its victory, D then filed suit against
P for failure to pay the final installment on the contract. Ds claim will be barred by the compulsory
counterclaim rule because it arose from the same transaction or occurrence as the first suit. Ds
claim will not be barred by claim preclusion because the parties are not positioned on the same side
of the v as in the first suit. Here, the defendant in the first suit has become a plaintiff.
2. Issue Preclusion
a. Defined
(1) While claim preclusion bars re-litigation of claims between the same parties, issue preclusion
(sometimes called collateral estoppel) bars the re-litigation of issueseven in cases not
involving the same parties. An issue is narrower than a claim. Put differently, the litigation of a
single claim may involve the determination of several different issues.
b. Elements
(1) For litigation of a particular factual issue to be precluded, four elements must be met:
(a) The issue must have been litigated and determined in the prior suit. It is not enough that an
issue was implicitly decided, stipulated to be true by the parties. The issue must be subject
to the judicial resolution, whether by the judge or jury.
(b) The issue must have been essential to the judgment. In practice, an issue is essential to
the judgment if it is a merits issue that if decided the opposite way, would change the result
of the case
EXAMPLE: P sues D for injuries she sustained when using Ds product. During discovery,
P sought a document from D that D claimed was privileged. P moved to compel and lost.
In a later suit, D claimed the document was privileged. The first courts determination of
privilege will not have any effect on the second suit because it did not pertain to the merits
of Ps claim.
EXAMPLE: P sued D for injuries she sustained on a bus trip. One issue in the case was
whether D was vicariously liable for the actions of the bus driver. The court held that it was,
and P later prevailed at trial. This issue was essential to judgment because it pertained to
the merits of Ps claim and, if decided differently, would have defeated Ps claim.
(c) The prior suit must have ended in a valid final judgment on the merits. The analysis here is
the same as that used in claim preclusion.
(d) The party against whom preclusion is asserted must have had a full and fair opportunity, as
well as incentive, to litigate the issue in the first suit. In practice, this often means that the
party defending itself from an issue preclusion argument was a party to the first suit
(though the party might still avoid issue preclusion if it lacked an incentive to litigate the
issue).
c. The importance of special verdicts
(1) Above, we considered juries and the types of verdicts they deliver. A general verdict is simply a
verdict that says P wins or D wins. In contrast, a special verdict says that P was contributory
negligent or D failed to warn the occupants of his store of a known risk. Special verdicts are
very helpful to parties asserting issue preclusion because they explicitly define the issue, how it
was decided, and (often) its essentiality to the judgment. Issue preclusion can be used without
special verdicts, but it is often easier to make the argument when the first case relied on them.
EXAMPLE: P is injured in a bus accident and, as above, sues the bus company on the theory that the
bus driver was negligent and the bus company is vicariously liable for the drivers negligence. The
jury returned a verdict for the bus company that specifically noted that the driver was not negligent. P
then filed suit against the driver. P is precluded from litigating the drivers negligence because that

41
issue was actually litigated and essential to judgment in the first suit, the first suit ended in a valid final
judgment on the merits, and P had a full and fair opportunity (and incentive) to litigate the bus drivers
negligence in the first suit. This type of preclusion is known as defensive issue preclusion (or
sometimes defensive collateral estoppel) because it is raised in defense to a claim.
EXAMPLE: As above, P is injured in a bus accident and sues the bus company using a vicarious
liability theory. She loses at trial but the special verdict form indicates only that her claim failed
because state law does not impose vicarious liability on companies operating as common carriers. P
then sues the bus driver for negligence. P is not precluded from litigating the drivers negligence
because his negligence was not essential to the judgment.
EXAMPLE: Again, P is injured in a bus accident. She sues the bus company, and the jury returns a
verdict in her favor, specifying that the driver was negligent and that the company is vicariously liable
for his negligence. After Ps victory, P2 (Ps friend who was also on the bus) sues the bus company.
The bus company will likely be precluded from contesting the drivers negligence or its vicarious
liability for that negligence. This is because: (1) the issues were actually litigated in the first suit, (2)
the issues were essential to the judgment, (3) the first case ended in a valid final judgment on the
merits, and (4) the bus company had a full and fair opportunity (and incentive) to litigate those issues
in the first suit. This type of preclusion is known as offensive issue preclusion (or offensive collateral
estoppel) because it is raised by a plaintiff pressing a claim.
NOTE: Courts are a bit more skeptical of this type of preclusion than the defensive version; they look
particularly closely at the defendants incentive to litigate the issues in the first suit. If the defendant
could not have reasonably known other plaintiffs would be waiting in the wings to piggy back on the
first plaintiffs successful verdict, the court will find that the defendant did not have an incentive to
vigorously defend itself and thus refuse to apply the rules of issue preclusion here.
EXAMPLE: P1 is injured in a bus accident. She sues D, the bus company, and loses in a jury trial.
The jury returned a special verdict stating that the accident was caused by another driver and neither
the bus company nor any of its agents caused the accident. Thereafter, P2 sues D for injuries she
sustained in the same accident. D moves to dismiss, arguing that P2 is preluded from litigating its
liability. The court should deny this motion because P2the party against whom preclusion is
asserteddid not have a full and fair opportunity to litigate the issue in the first suit. In fact, she
wasnt even a party to the first suit!

42

You might also like